Quick viewing(Text Mode)

IAS 2020 Prelims TEST SERIES Test - 21 HISTORY - 3 Ancient + Medieval India

IAS 2020 Prelims TEST SERIES Test - 21 HISTORY - 3 Ancient + Medieval India

https://t.me/UPSC_PDF Website ➡ https://upscpdf.com https://t.me/UPSC_PDF

IAS 2020 Prelims TEST SERIES Test - 21 HISTORY - 3 Ancient + Medieval

www.iasscore.in

Website ➡ https://upscpdf.com https://t.me/UPSC_PDF Website ➡ https://upscpdf.com https://t.me/UPSC_PDF

DO NOT OPEN THIS TEST BOOKLET UNTIL YOU ARE ASKED TO DO SO

PTS2020/PPP-21/122019/26

GENERAL STUDIES Test No. HISTORY - 3 Ancient + Medieval India 21

Time Allowed: Two Hours Maximum Marks: 200

I N S T R U C T I O N S

1. IMMEDIATELY AFTER THE COMMENCEMENT OF THE EXAMINATION, YOU SHOULD CHECK THAT THIS TEST BOOKLET DOES NOT HAVE ANY UNPRINTED OR TORN OR MISSING PAGES OR ITEMS, ETC. IF SO, GET IT REPLACED BY A COMPLETE TEST BOOKLET. 2. Please note that it is the candidate’s responsibility to encode and fi ll in the Roll Number carefully without any omission or discrepancy at the appropriate places in the OMR Answer Sheet. Any omission/discrepancy will render the Answer Sheet liable for rejection. 3. You have to enter your Roll Number on the test booklet in the Box provided alongside. DO NOT write anything else on the Test Booklet. 4. This Test Booklet contains 100 items (questions). Each item is printed in English. Each item comprises four responses (answers). You will select the response which you want to mark on the Answer Sheet. In case you feel that there is more than one correct response, mark the response which you consider the best. In any case, choose ONLY ONE response for each item. 5. You have to mark all your responses ONLY on the separate Answer Sheet Provided. See directions in the Answer Sheet. 6. All items carry equal marks. 7. Before you proceed to mark in the Answer Sheet in response to various items in the Test Booklet, you have to fi ll in some particulars in the Answer Sheet as per instructions sent to you with your Admission Certifi cate. 8. After you have completed fi lling in all your responses on the Answer Sheet and the examination has concluded, you should hand over to the Invigilator only the Answer Sheet. You are permitted to take away with you the Test Booklet. 9. Sheets for rough work areGS appended in the Test BookletSCORE at the end. 10. Penalty for wrong answers: THERE WILL BE PENALTY FOR WRONG ANSWERS MARKED BY A CANDIDATE IN THE OBJECTIVE TYPE QUESTION PAPERS. (i) There are four alternatives for the answer to every question. For each question for which a wrong answer has been given by the candidate, one-third of the marks assigned to that question will be deducted as penalty. (ii) If a candidate gives more than one answer, it will be treated as a wrong answer even if one of the given answers happens to be correct and there will be same penalty as above to that question. (iii) If a question is left blank, i.e., no answer is given by the candidate, there will be no penalty for that question.

DO NOT OPEN THIS TEST BOOKLET UNTIL YOU ARE ASKED TO DO SO 1 PTS2020/PPP-21/122019/26

PTS2020/PPP-21/122019/26

Website ➡ https://upscpdf.com https://t.me/UPSC_PDF Website ➡ https://upscpdf.com https://t.me/UPSC_PDF

1. With reference to the Stone Age culture in 3. Muzarain were the tenants who paid land India, consider the following statements: revenue at higher rates as compared to other classes of peasants. 1. The lower Paleolithic tool technology is characterized by flake tool industry. Which of the above statements is/are incorrect? 2. Middle Paleolithic tools have mostly been found in Northern India. (a) 1 only 3. Microliths are the tools of Mesolithic age (b) 1 and 2 only characterized by parallel-side blades. (c) 2 and 3 only Which of the above statements is/are correct? (d) None of the above (a) 1 and 2 only (b) 2 only 5. The ending of Gupta era coincided with rise of various small and large states in north-eastern (c) 3 only region. In this context, consider the following (d) 1 and 3 only statements: 1. Gauda was the region situated between 2. With reference to the nobility under the the Ganga and Brahmaputra. Mughals consider the following statements: 2. Dandabhakti was a fiscal and 1. Nobility’s aristocratic character did administrative unit formed is border areas between Bengal and Orissa. not provide avenues of promotion and distinction to lower sections of society. Which of the above statements is/are correct? 2. The Marathas were ousted from nobility (a) 1 only during the times of Aurangzeb. (b) 2 only Which of the above statements is/are correct? (c) Both 1 and 2 (a) 1 only (d) Neither 1 nor 2 (b) 2 only 6. Arrange the following dynasties of Vijaynagar (c) Both 1 and 2 Empire in chronological order: (d) Neither 1 nor 2 (a) Sangama - Tuluva - Saluva - Aravidu (b) Sangama - Saluva - Tuluva - Aravidu 3. Consider the following statements regarding Ashokan Inscriptions: (c) Sangama - Aravidu - Tuluva - Saluva 1. Ashoka’s name does not occur in any (d) Saluva - Aravidu - Sangama - Tuluva of his inscriptions and mentioned as ‘devanampiya piyadasi’. 7. Match the following: 2. The title devanampiya or ‘dear to gods’ Vedas Characteristic adopted by Ashoka was unique and was not adopted by his ancestors. A. Rigveda 1. Collection of sacrificial formulae Which of the above statements is/are correct? B. Samveda 2. Collection of hymns (a) 1 only GS SCORE C. Yajurveda 3. Collection of spells and (b) 2 only charms (c) Both 1 and 2 D. Atharvaveda 4. Collection of songs (d) Neither 1 nor 2 Select the correct answer using the code given below: 4. Consider the following statements regarding A B C D the village life under Mughal rule: (a) 1 3 4 2 1. Kamins were the landless peasants who (b) 4 1 2 3 often belonged to the untouchable class. (c) 2 4 1 3 2. Khudkasht were the peasants who owned the land they tilled. (d) 3 2 4 1

PTS2020/PPP-21/122019/26 2

Website ➡ https://upscpdf.com https://t.me/UPSC_PDF Website ➡ https://upscpdf.com https://t.me/UPSC_PDF

8. Which of the following are the significances of (c) 2 and 3 only ’s advent into India? (d) 1, 2, 3 and 4 1. Security to India from external invasions 2. Introduction of arch and dome in region’s 11. The Maurya period constitutes a landmark in architecture the system of taxation in ancient India. In this 3. Introduction and popularization of context, consider the following statements: gunpowder and artillery in India 1. The sannidhata was the highest officer in 4. Establishment of tradition of garden charge of assessment and collection. building 2. The samaharta was the chief custodian Select the correct answer using the code given of the state treasury and storehouse. below: 3. The assessor was far more important (a) 1, 2 and 3 only than the chief treasurer. (b) 2 and 4 only Which of the above statements is/are correct? (c) 1 and 4 only (a) 1 and 2 only (d) 1, 2, 3 and 4 (b) 2 and 3 only (c) 3 only 9. Match the following: (d) 1, 2 and 3 Important Archeological Findings Harappan 12. Consider the following statements regarding Sites the organization of central and provincial A. Harappa 1. Remains of horse bones governments under Akbar: 1. Akbar organized the central B. Banawali 2. Red sandstone male torso administration on basis of division of C. Ropar 3. City with radial streets powers between various departments. D. Surkotada 4. Dog buried with human 2. Wazir was considered to be the head of nobility. Select the correct answer using the code given below: 3. The provincial administration was organized on the model of central A B C D administration. (a) 1 4 3 2 Which of the above statements is/are correct? (b) 2 3 4 1 (a) 1 and 2 only (c) 3 1 2 4 (b) 1 and 3 only (d) 4 2 1 3 (c) 2 only (d) 3 only 10. ‘Mohammad Bin Tughlaq was known as an ill-fated idealist’. In this context consider 13. Consider the following statements regarding the following events that occurred during his the later Vedic polity: reign: GS SCORE1. It was a monarchical system. 1. He is known to have transferred his 2. It gave greater stress on hereditary capital from to Devagiri. rulership. 2. He issued copper coins at par with the 3. Vidatha replaced Sabha and Samiti as value of the silver tanka coins. the popular assembly to check the king’s 3. Ibn Batuta acted as a Qazi in Delhi for authority. eight years during his reign. Which of the above statements is/are correct? 4. Fort of Adilabad was built under his rule. (a) 1 and 2 only Which of the above statements are correct? (b) 2 only (a) 1, 2 and 4 only (c) 3 only (b) 1 and 3 only (d) 1 and 3 only

3 PTS2020/PPP-21/122019/26

Website ➡ https://upscpdf.com https://t.me/UPSC_PDF Website ➡ https://upscpdf.com https://t.me/UPSC_PDF

14. Consider the following statements with respect 2. Financial Crisis in the empire due to to the Lodhi Dynasty in India: payment to Bureaucracy 1. Bahlol Lodhi laid the foundation of the 3. Neglect of the North-West Frontier and Lodhi dynasty in India. the creation of Great Wall of China 2. Sikandar Lodi wrote Persian verses 4. Oppressive Rule of Bureaucrats under the name Gulrakhi Select the correct answer using the code given Which of the above statements is/are correct? below: (a) 1 only (a) 1 and 4 only (b) 2 only (b) 2 and 3 only (c) Both 1 and 2 (c) 1, 2 and 4 only (d) Neither 1 nor 2 (d) 1, 2, 3 and 4 15. Consider the following statements regarding pre-historic paintings: 18. Qutub-ud-din Aibak was the founder of the 1. The pre-historic paintings were generally Slave Dynasty in India. In this context, executed on rocks. consider the following statements: 2. The most common scene depicted in these 1. The first mosque in India was built by paintings is of group hunting. him. 3. Animals are only drawn in bold outline 2. He is known to have issued a large while human bodies are filled in number of silver coins in India. completely. 3. He started the construction of the Qutub 4. Bhimbetka in Madhya Pradesh is an Minar in Delhi. important site for these paintings. Which of the above statements are correct? Which of the above statements is/are correct? (a) 1 and 3 only (a) 1 and 2 only (b) 1, 2 and 4 only (b) 1 and 3 only (c) 2 and 4 only (c) 2 and 3 only (d) 1, 2, 3 and 4 (d) 1, 2 and 3

16. With reference to the religious policy of Akbar, 19. Which of the following are the traits linked to consider the following statements: Aryan culture? 1. Akbar’s attitude towards his Hindu 1. Use of horse-drawn chariots subjects was guided by the policy of Sulh- i-kul. 2. Practice of pit dwelling 2. The proceedings at Ibadat Khana were 3. Practice of cremating the dead initially confined to Muslims only. 4. Horse sacrifice 3. Akbar propounded Tauhid-i-Ilahi which Select the correct answer using the code given combined elements from other existing below: religions. GS SCORE Which of the above statements is/are correct? (a) 1 only (a) 1 only (b) 2 and 3 only (b) 1 and 2 only (c) 1, 2 and 3 only (c) 2 and 3 only (d) 1, 2, 3 and 4 (d) 1, 2 and 3 20. Consider the following statements: 17. Which of the following were the causes of the 1. The first Battle of Panipat was fought fall of Mauryan Empire? between Babur and Rana Sanga in 1526. 1. Introduction of Vyavaharasamata and 2. The Battle of Khanwa was fought between Dandasamata Babur and Ibrahim Lodi in 1527.

PTS2020/PPP-21/122019/26 4

Website ➡ https://upscpdf.com https://t.me/UPSC_PDF Website ➡ https://upscpdf.com https://t.me/UPSC_PDF

3. The Battle of Khanwa was more decisive 24. Which among the following Mughal Emperor than the Battle of Panipat as it secured built a new city – ‘Dinpanah’ at Delhi? Babur’s position in the Delhi-Agra region. (a) Babur Which of the above statements is/are correct? (b) Humayun (a) 1 and 2 only (c) Akbar (b) 3 only (d) Jahangir (c) 1, 2 and 3 25. Consider the following statements regarding (d) None of the above Early Rig Vedic Period: 1. The Rig Vedic people were mainly 21. Which one of the following statements is pastoral and thus fought wars for the incorrect with respect to Gupta ruler, sake of cows. Chandragupta II? 2. Dasas and Dasyus were slaves and (a) The Gupta Empire was at its peak during Shudras who were conquered by Aryans. the reign of Chandragupta II. Which of the above statements is/are correct? (b) Kalidasa and Amarasimha were scholars at the court of Chandragupta II at (a) 1 only Ujjain. (b) 2 only (c) Chinese pilgrim Fa-hsien visited India during his reign. (c) Both 1 and 2 (d) Chandragupta II started the Vikrama (d) Neither 1 nor 2 samvat or era. 26. Match the following: 22. Arrange the following rulers of Slave Dynasty List-I List-II in the chronological order: A. Jalauddin Khalji 1. Deposed Malik 1. Bahram Shah Kafur 2. Nasiruddin Mahmud B. Alauddin Khalji 2. Settled New 3. Balban Musalmans near 4. Alauddin Masud Shah Delhi Select the correct answer using the code given C. Qutbuddin 3. Only Hindu convert below: Mubarak Shah to sit on the throne of Delhi (a) 1-2-3-4 (b) 3-1-4-2 D. Nasiruddin 4. Renamed Chittor to Khusrau Khizrabad (c) 1-4-2-3 Select the correct answer using the code given (d) 3-2-4-1 below: A B C D 23. He was the most famous Chalukya king. He subjugated the KadambaGS capital at Banavasi SCORE (a) 2 4 1 3 and compelled the Gangas of Mysore to acknowledge his suzerainty. He also conquered (b) 3 4 1 2 the entire area between the Krishna and the (c) 4 3 2 1 Godavari, which came to be known as the province of Vengi. (d) 3 2 1 4 Which of the following rulers is described in above passage? 27. Consider the following statements: (a) Narshimhavarman 1. The Harappan Civilization marked the beginning of the second urbanization in (b) Pushyamitra Shunga India. (c) Vikramditya II 2. Harappan people did not practice (d) Pulakeshin II ploughing as iron was unknown to them.

5 PTS2020/PPP-21/122019/26

Website ➡ https://upscpdf.com https://t.me/UPSC_PDF Website ➡ https://upscpdf.com https://t.me/UPSC_PDF

3. Animals depicted on the famous (c) Muhammad Quli Qutub Shah Pashupati seals excavated at Harappa are a rhino, a buffalo, an elephant and (d) Yusuf Adil Shah a tiger. Which of the above statements is/are 31. Consider the following statements: incorrect? 1. Rig Vedic tribal chiefs had unlimited (a) 1 and 2 only powers. (b) 2 only 2. Vrajapati was the officer responsible for collection of taxes in the Rig Vedic (c) 2 and 3 only period. (d) 1, 2 and 3 3. Status of women in the early Vedic period was better than that in later Vedic 28. The customs “Sijada” and “Paibos” were period. introduced by which of the following ruler? Which of the above statements is/are (a) Bahram Shah incorrect? (b) Nasiruddin Mahmud (a) 1 only (c) Alauddin Masud Shah (b) 1 and 2 only (d) None of the above (c) 2 and 3 only (d) 3 only 29. Consider the following statements regarding spread of material culture during Mauryan 32. Akbar introduced a number of social and Age: educational reforms. Which of the following 1. This period provide evidences for use of reforms were introduced by him in this socketed axes, hoes, spades, sickles, and regard? ploughshares. 1. Legalized widow remarriage 2. Ring wells were first constructed under the Mauryas in the Gangetic plains. 2. Raised marriage age for both boys and 3. Mauryan construction does not give any girls evidence of use of burnt bricks. 3. Complete ban on practice of sati Which of the above statements are correct? 4. Introduction of secular subjects (a) 1 and 2 only Select the correct answer using the code given (b) 1 and 3 only below: (c) 2 and 3 only (a) 1 and 3 only (d) 1, 2 and 3 (b) 1, 2 and 4 only

30. He was a King of Sultanate of Bijapur. He was (c) 2, 3 and 4 only very solicitous to the poor and had the title of ‘abla baba’ i.e. ‘Friend of Poor’. He was deeply (d) 1, 2, 3 and 4 interested in musicGS and composed a SCOREbook called ‘Kitab-i-Nauras’ in which songs were set 33. Match the following: to various musical modes. He also built a new capital ‘Nauraspur’ in which a large number of Terminology Feature musicians were invited to settle. He accorded A. Vishti 1. Book on astronomy patronage to all, including Hindu saints and temples and due to his broad approach; he B. Kumaramatyas 2. Part of provincial came to be called ‘Jagat Guru’. and local administration Identify the personality described in the above system passage? C. Vishayas 3. Important officers (a) Adil Ali Shah in the Gupta empire (b) Ibrahim Adil Shah II D. Romaka Sidhanta 4. Forced labour

PTS2020/PPP-21/122019/26 6

Website ➡ https://upscpdf.com https://t.me/UPSC_PDF Website ➡ https://upscpdf.com https://t.me/UPSC_PDF

Select the correct answer using the code given Select the correct answer using the code given below: below: A B C D A B C D (a) 4 2 3 1 (a) 2 3 4 1 (b) 2 4 1 3 (b) 4 2 1 3 (c) 4 3 2 1 (c) 2 4 3 1 (d) 3 4 1 2 (d) 4 1 3 2

34. Consider the following statements with 37. With reference to the Rudradaman I, consider respect to the socio-economic system during the following statements: the Sultanate period: 1. He was the most famous Kushan ruler 1. Khalisa was the land under the direct who ruled over Sindh. control of the Sultan. 2. He undertook the repair works to improve 2. Zimmis were the Turkish nobles who the Sudarshana Lake in the semi-arid were positioned at high places in the zone of Kathiawar. Sultanate rule. 3. He was a great lover of Sanskrit. Which of the above statements is/are correct? Which of the above statements is/are (a) 1 only incorrect? (b) 2 only (a) 1 and 2 only (c) Both 1 and 2 (b) 1 only (d) Neither 1 nor 2 (c) 2 and 3 only

35. Which of the following statements regarding (d) None of the above the Neolithic Age is/are correct? 1. The earliest Neolithic settlements in the 38. Consider the following statements regarding developed in the Maratha ruler Shivaji’s organisation of the west of River Indus. army and the revenue system: 2. The Kashmiri Neolithic culture was 1. Regular soldiers were given salaries in characterized by the complete absence of cash. microliths. 2. The revenue system was patterned on 3. Pottery first appeared in this phase. the system of Malik Ambar. 4. People in this phase found settlements 3. Shivaji abolished the Zamindari system. away from the hilly areas. Which of the above statements is/are correct? Select the correct answer using the code given (a) 1 only below: (b) 1 and 2 only (a) 1, 2 and 3 only (c) 3 only (b) 2 and 4 only GS SCORE(d) 1, 2 and 3 (c) 3 only (d) 1, 3 and 4 only 39. Consider the following statements regarding religious practices during the Rig Vedic 36. Match the following listed taxes imposed by period: Firoz Shah Tughlaq: 1. There was no idol worship during this Tax Purpose period. A. Sharb 1. Imposed on 10% of produce 2. The dominant mode of worshipping was recitation of prayers and performance of B. Kharaj 2. 20% spoils of war sacrifices accompanied by rituals. C. Zakat 3. Alms for Muslims 3. Female gods were less important than D. Khams 4. Irrigation tax male gods.

7 PTS2020/PPP-21/122019/26

Website ➡ https://upscpdf.com https://t.me/UPSC_PDF Website ➡ https://upscpdf.com https://t.me/UPSC_PDF

Which of the above statements are correct? Select the correct answer using the code given below: (a) 1 and 2 only (a) 1 only (b) 1 and 3 only (b) 2 only (c) 2 and 3 only (c) Both 1 and 2 (d) None of the above (d) Neither 1 nor 2

40. Match the following literatures related to the 43. The Gupta period is remarkable for the period with their Authors: production of literary works. In this context List I List II consider the following statements: A. Kitab-ul-Hind 1. Alberuni 1. Bhasa was the author of a drama called Dradiracharudatta. B. Khazain-ul-Futuh 2. Amir Khusrau 2. Amarakosha was compiled by C. Tarikh-i-Firoz Shahi 3. Barani Amarasimha. D. Tabaqat-i-Nasari 4. Minhaj-us-Siraj Which of the above statements is/are correct? Select the correct answer using the code given (a) 1 only below: (b) 2 only A B C D (c) Both 1 and 2 (a) 1 2 3 4 (d) Neither 1 nor 2 (b) 3 4 1 2 44. With reference to Sher Shah Suri, the (c) 1 3 2 4 founder of , consider the following (d) 4 2 3 1 statements: 1. He made significant changes in the administration divisions as compared 41. With respect to the Gupta administration, to Delhi Sultanate by following a consider the following statements: decentralized system of administration. 1. Absence of hereditary succession in high- 2. He introduced liberal policies like ranking officials. abolition of Jizyah. 2. Grant of fiscal and administrative 3. He paid great attention to improvement concessions to priests and of communications in his kingdom. administrators. Which of the above statements is/are 3. Both Visyapati and Uparika were incorrect? appointed directly by the Gupta (a) 1 and 2 only emperor. (b) 2 only Which of the above statements is/are correct? (c) 2 and 3 only (a) 1 and 2 only (d) None of the above (b) 2 only GS SCORE (c) 1 and 3 only 45. Which of the following statements regarding Harappan Civilization is/are correct? (d) 2 and 3 only 1. Harappans were both phallus and animal worshipers. 42. Which of the following was/were the features of 2. Harappan script is largely pictographic Inam grants made during the reign of Akbar? which has not been deciphered yet. 1. Inam lands were granted to all persons Select the correct answer using the code given irrespective of their religious faiths and below: beliefs. (a) 1 only 2. Half of the inam lands consisted of cultivable wasteland. (b) 2 only

PTS2020/PPP-21/122019/26 8

Website ➡ https://upscpdf.com https://t.me/UPSC_PDF Website ➡ https://upscpdf.com https://t.me/UPSC_PDF

(c) Both 1 and 2 3. Kharoshthi (d) Neither 1 nor 2 4. Brahmi Select the correct answer using the code given 46. With reference to Dahsala or Zabti system below: of land revenue assessment, consider the (a) 1 and 4 only following statements: 1. It was a system of ten year settlement (b) 2 and 4 only but not a permanent one as the state (c) 2, 3 and 4 only retained the right of modification. (d) 1, 2, 3 and 4 2. Peasants were given the choice of paying in cash or kind under this system . 50. Consider the following statements regarding Which of the above statements is/are correct? organization of the local government during (a) 1 only Akbar’s reign: (b) 2 only 1. Amalguzar was the officer-in-charge of law and order in a pargana. (c) Both 1 and 2 2. Inam lands were those which were (d) Neither 1 nor 2 allotted to learned and religious men. Which of the above statements is/are correct? 47. They were nomadic people from the steppes of North- living in the neighbourhood (a) 1 only of China, first occupied or north (b) 2 only Afghanistan and gradually moved to the Kabul valley and seized Gandhara by crossing the (c) Both 1 and 2 Hindu Kush. Their empire extended from the (d) Neither 1 nor 2 Oxus to the Ganges, from Khorasan in Central Asia to Pataliputra in Bihar. 51. Consider the following statements regarding Which of the following dynasties is described Chalcolithic age: in above passage? 1. Chalcolithic people were primarily rural (a) Shakas communities who used only copper (b) Kushans objects. 2. Both black-and-red pottery and ochre- (c) Parthians colored pottery are associated with this (d) Indo-Greeks phase. 3. The settlement pattern and burial 48. Arrange the following rulers of Saiyyad practices suggest the beginnings of social Dynasty in chronological order: inequalities in Chalcolithic society. (a) Khizr Khan - Mubarak Shah - Muhammad Which of the above statements is/are Shah - Alauddin Alam Shah incorrect? (b) Mubarak ShahGS - Alauddin Alam ShahSCORE - (a) 1 only Khizr Khan - Muhammad Shah (b) 1 and 2 only (c) Khizr Khan - Muhammad Shah - Alauddin Alam Shah - Mubarak Shah (c) 2 and 3 only (d) Alauddin Alam Shah - Khizr Khan - (d) 1, 2 and 3 Mubarak Shah - Muhammad Shah 52. Consider the following statements regarding 49. Ashokan inscriptions, considered to be the Turkish king Balban: earliest were written in which of the following 1. The Persian festival of Nauroz was scripts? introduced by him. 1. Aramaic 2. He is known to have included non-Turks 2. Greek to his administration.

9 PTS2020/PPP-21/122019/26

Website ➡ https://upscpdf.com https://t.me/UPSC_PDF Website ➡ https://upscpdf.com https://t.me/UPSC_PDF

3. Balban followed the policy of ‘Tolerance Select the correct answer using the code given and Recognition’ for building peace and below: control in the region. A B C D Which of the above statements is/are (a) 2 3 4 1 incorrect? (b) 1 4 3 2 (a) 1 and 2 only (c) 3 4 1 2 (b) 2 and 3 only (d) 4 1 3 2 (c) 3 only (d) 1, 2 and 3 56. Which of the following statements is incorrect with respect to the Delhi Sultanate art and 53. Which of the following were the consequences architecture? of Central Asian Invasion in India? (a) Alai Darwaza at the entrance of Qutub 1. Better Cavalry Minar was built by Qutbuddin Mubarak Shah. 2. Promotion of Agriculture (b) Tomb of Ghyasuddin was built by 3. Introduction of Gold coins on a wide Muhammad bin Tughlaq. scale. (c) Kotla fort at Delhi was the creation of 4. Origin of Hinayana Buddhism Firoz Tughlaq. Select the correct answer using the code given (d) The palace complex called Tughlaqabad below: was built during the period of Ghyasuddin (a) 1, 2 and 3 only Tughlaq. (b) 2 and 3 only 57. With reference to the Art and Architecture (c) 1 and 4 only during the Gupta Period, consider the following statements: (d) 1, 2, 3 and 4 1. The gold coin issued by Samudragupta shows him playing the vina. 54. Consider the following statements regarding the Chola Government: 2. Gupta period was rich in terms of the architectural developments. 1. All the authority rested in King’s hand with Council of Ministers to advise him. 3. Ajanta paintings were built during the Gupta period. 2. The basic unit of administration was ‘Nadu’ which consisted of a number of Which of the above statements are correct? villages. (a) 1 and 2 only Which of the above statements is/are correct? (b) 1 and 3 only (a) 1 only (c) 2 and 3 only (b) 2 only (d) 1, 2 and 3 (c) Both 1 and 2 GS SCORE (d) Neither 1 nor 2 58. With reference to the cultural contributions of the Deccan States, consider the following statements: 55. Match the following: 1. Ali Adil Shah invited Christian Books Writers missionaries to his court even before A. Malavikagnimitram 1. Bilhana Akbar had done so. 2. Muhammad Quli Qutub Shah was a B. Mudrarakshasha 2. Banabhatta contemporary of Akbar who constructed C. Vikramankadevacharita 3. Kalidasa Char Minar. D. Harshacharita 4. Vishakha- 3. Gol Gumbaz built by rulers of Bijapur has datta the single largest dome ever constructed.

PTS2020/PPP-21/122019/26 10

Website ➡ https://upscpdf.com https://t.me/UPSC_PDF Website ➡ https://upscpdf.com https://t.me/UPSC_PDF

Which of the above statements is/are correct? 1. Nayaks were army officers who were (a) 1 only granted land in lieu of their services. 2. The practices of devadasi and sati were (b) 1 and 2 only absent during this period. (c) 2 and 3 only Which of the above statements is/are correct? (d) 1, 2 and 3 (a) 1 only (b) 2 only 59. Consider the following statements regarding Kalabhra Revolt: (c) Both 1 and 2 1. The revolt was directed against the (d) Neither 1 nor 2 existing social and political order in south India. 63. Match the following: 2. They captured power at the cost of the List-I List-II Cholas. A. Mahamatras 1. High Officials of 3. They were supressed through the joint administrative efforts of the Pandyas, the Pallavas, and division the Chalukyas. B. Gaulmika 2. Provincial Governor Select the correct answer using the code given below: C. Maharashtrikas 3. Head of Military Regiment (a) 1 and 2 only D. Senapati 4. District Officials (b) 2 and 3 only Select the correct answer using the code given (c) 1 and 3 only below: (d) 1, 2 and 3 A B C D (a) 3 4 1 2 60. Jalaluddin Khalji laid the foundation of the Khalji Dynasty in India. In this context, (b) 3 4 2 1 consider the following statements: (c) 4 3 1 2 1. He is known to have a benevolent attitude towards . (d) 4 3 2 1 2. He maintained the policy of tolerance to win the goodwill of nobility. 64. In fixing the land revenue during the reign Which of the above statements is/are correct? of Akbar, continuity of cultivation was taken into account and land was divided into (a) 1 only four categories. In this context, match the (b) 2 only following: (c) Both 1 and 2 List I List II (d) Neither 1 nor 2 A. Polaj 1. Land uncultivated for 5 years or more 61. Which one of the following statements is B. Parati 2. Land which is temporarily incorrect with respect to social developments uncultivated in the Gupta period? GS SCOREC. Chachar 3. Land always under (a) The position of Shudras improved during cultivation this period. D. Banjar 4. Land left fallow for 2-3 (b) Gupta period is marked with the years subordination of women of upper varnas. Select the correct answer using the code given (c) Brahmans became direct tillers of the below: soil due to the various land grants. A B C D (d) Daughters were not allowed to inherit landed property due to patriarchal (a) 2 1 4 3 setup. (b) 3 2 4 1 (c) 4 2 3 1 62. Consider the following statements with respect to social life during the Vijayanagar period: (d) 2 4 3 1

11 PTS2020/PPP-21/122019/26

Website ➡ https://upscpdf.com https://t.me/UPSC_PDF Website ➡ https://upscpdf.com https://t.me/UPSC_PDF

65. Large scale human settlements generally (c) Both 1 and 2 spread from west to east in the Indian sub- (d) Neither 1 nor 2 continent. What can be the possible reasons for this? 68. The death of Iltutmish created a political 1. Presence of various passes in north-west turmoil for accession to the throne. In this mountain ranges context, consider the following statements: 2. Easier clearance of forests in the 1. Razia Sultan seized the throne of the Himalayan foothills Delhi Sultanate from Bahram Shah. 3. Narrower width of rivers in the 2. Malik Jamaluddin Yaqut is known to Himalayan foothills have imprisoned Razia at Mubarak 4. Gradual increase in rainfall from west to in . east Which of the above statements is/are correct? Select the correct answer using the code given (a) 1 only below: (b) 2 only (a) 1 only (c) Both 1 and 2 (b) 1 and 4 only (d) Neither 1 nor 2 (c) 2 and 3 only (d) 1, 2, 3 and 4 69. With reference to sculptures of Indus Valley/ Harappan Civilization, consider the following statements: 66. Match the following listed items given below: 1. All of the Harappan seals were square in List I List II shape and were made of steatite. A. Ghiyasuddin 1. Safarnama Rehla 2. The terracotta sculptures are more in Tughlaq number as compared to bronze figures. B. Muhammad 2. A Telugu Brahmin 3. Northern Black Polished Ware was bin Tughluq convert Wazir associated with the Indus Valley Civilization. C. Ibn Batuta 3. First sultan to start irrigation works 4. Bones and baked clay were used to make ornaments by the Harappans. D. Khan-i-Jahan 4 Diwan-i-amirkohi Maqbal Which of the above statements is/are incorrect? Select the correct answer using the code given below: (a) 1 only A B C D (b) 2 and 4 only (a) 2 3 4 1 (c) 1, 2 and 3 only (b) 3 4 1 2 (d) 3 only (c) 4 3 2 1 70. Consider the following statements regarding (d) 3 4 1 2 the Mansabdari system: GS SCORE1. Under this system, every officer was 67. Consider the following statements with respect assigned a mansab which was divided to Ashoka’s edicts and inscriptions: into zat and sawar. 1. There is no reference to the teachings of 2. Princes of blood received higher the Buddha in the Greek and Aramaic mansabs. versions of his inscriptions. 3. Mansabdars were either assigned a jagir 2. The inscriptions throw light on his or paid in cash. external and domestic polices as well as 4. The system served the dual objective of the extent of his empire. organizing the nobility as well the army. Which of the above statements is/are correct? Which of the above statements are correct? (a) 1 only (a) 1 and 3 only (b) 2 only (b) 2 and 4 only

PTS2020/PPP-21/122019/26 12

Website ➡ https://upscpdf.com https://t.me/UPSC_PDF Website ➡ https://upscpdf.com https://t.me/UPSC_PDF

(c) 2, 3 and 4 only (b) Mark the horses of sawars (d) 1, 2, 3 and 4 (c) Mark the swords of each soldier (d) Mark the divisions of land for revenue 71. With respect to ‘Megaliths’ in the Indian assessment historical period, consider the following statements: 75. He defeated the Shakas and destroyed many 1. Megaliths were people living in the deltaic kshatriya rulers. His empire extended from region of the deep southern peninsular Malwa in the north to Karnataka in the south. India as early as 1000 BC. The fortunes of the Satavahanas were restored 2. They were predominantly agriculturists by him. He called himself the only brahmana. with less advanced types of agricultural Identify the ruler described in above passage? tools. 3. The social arrangement developed within (a) Rudradaman-I their primitive societal setup is known as (b) Gautamiputra Satakarni Tamizhakam. (c) Vashishthiputra Pulumayi Which of the above statements is/are correct? (d) Yajna Sri Satakarni (a) 1 and 2 only

(b) 2 and 3 only 76. Arrange the following battles fought by (c) 1 and 3 only Mughals for establishing their empire in the correct chronological order? (d) None of the above 1. Battle of Khanwa 72. Which of the following statements is incorrect 2. Battle of Sirhind regarding Alauddin Khalji? 3. Battle of Chausa (a) He is known to have refuted the 4. First Battle of Panipat suzerainty of the Caliph. Select the correct answer using the code given (b) Alauddin was the first ruler to fix land below: revenue in cash. (a) 1-4-2-3 (c) The title of Tuti-i-Hind was given to him by Amir Khusrau. (b) 1-4-3-2 (d) He followed the policy of ruthless (c) 4-1-2-3 governance to maintain his control over (d) 4-1-3-2 the state. 77. With reference to Ancient History of India, the 73. Consider the following statements with respect term ‘Vaniggrama’ refers to: to religious developments during the Gupta Period: (a) Coastal cities. 1. The Gupta kings followed a policy of (b) Group of merchants tolerance towards different religious sects. (c) Village of merchants 2. Bhagavatism or Vaishnavism (d) Brahmana settlements overshadowed Mahayana Buddhism during Gupta times.GS SCORE 78. Match the following listed items with respect Which of the above statements is/are correct? to central administration during the Delhi Sultanate period: (a) 1 only List I List II (b) 2 only A. Diwan-i-Wizarat 1. Department of (c) Both 1 and 2 correspondence (d) Neither 1 nor 2 B. Diwan-i-Arz 2. Department of religious affairs 74. The ‘Dagh System’ during Akbar’s reign was employed to: C. Diwan-i-Rasalat 3. Military department (a) Mark territorial boundaries of D. Diwan-i-Insha 4. Finance department

13 PTS2020/PPP-21/122019/26

Website ➡ https://upscpdf.com https://t.me/UPSC_PDF Website ➡ https://upscpdf.com https://t.me/UPSC_PDF

Select the correct answer using the code given 82. A number of systems for land revenue below: assessments were followed during the reign of Akbar. In this context, consider the following A B C D statements: (a) 4 3 2 1 1. Under the Dahsala system, both the (b) 1 4 3 2 average produce and prices prevailing (c) 1 3 4 2 over the last 10 years of crops were calculated. (d) 4 1 2 3 2. Under the Nasaq system, the produce was divided between the peasants and 79. ‘The southern end of the Indian peninsula state in fixed proportion. situated south of the Krishna River was 3. Under the Batai system, the amount divided into three kingdoms: Chola, Pandya, payable was calculated based on an and Chera’. In this context, consider the assessment of what they have been following statements. paying in the past. 1. The city of Puhar was founded by the Which of the above statements is/are correct? Chola king Karikala. 2. Senguttuvan was a Chera king who (a) 1 only invaded the north and crossed the (b) 1 and 2 only Ganges. (c) 2 and 3 only Which of the above statements is/are correct? (d) 1, 2 and 3 (a) 1 only (b) 2 only 83. Consider the following statements regarding family structure during Satavahanas phase: (c) Both 1 and 2 1. The Satavahanas show traces of a (d) None of the above matrilineal social structure. 2. Succession to the throne was passed to 80. Who among the following is known to have the male member. acquired the title of Gajabetegara? Which of the above statements is/are correct? (a) Deva Raya II (a) 1 only (b) Harihara II (b) 2 only (c) Krishna Deva Raya (c) Both 1 and 2 (d) None of the above (d) Neither 1 nor 2

81. Nearly 1400 settlements of the Harappan Civilization have been discovered so far and 84. With reference to Iltutmish, which of the following statements is/are incorrect? these are distributed over a wide geographical area. In this context, consider the following 1. He shifted the capital from Lahore to statements: Delhi. 1. The Harappan Civilization extended 2. He was the first to issue purely Arabic from Jammu &GS Kashmir in the northSCORE to coinage of gold tanka. Maharashtra in the south. 3. He disposed of Aram Shah and became the 2. Majority of the settlements are located Sultan with the name of Shamsuddin. on the banks of river Indus and its 4. Turkan-i-chahalgani was organized by tributaries. him for administering the sultanate. Which of the above statements is/are Select the correct answer using the code given incorrect? below: (a) 1 only (a) 1 and 2 only (b) 2 only (b) 2 only (c) Both 1 and 2 (c) 3 and 4 only (d) Neither 1 nor 2 (d) None of the above

PTS2020/PPP-21/122019/26 14

Website ➡ https://upscpdf.com https://t.me/UPSC_PDF Website ➡ https://upscpdf.com https://t.me/UPSC_PDF

85. Match the following listed items with respect 3. His southward march was stopped by the to Sangam Literature: Chalukya king Pulakeshin. List I List II Which of the above statements is/are A. Arasar 1. Love story of incorrect? Kovalan and a (a) 1 only courtesan Madhavi of Kaveripattanam (b) 1 and 2 only B. Melkannakku 2. Ruling class (c) 3 only C. Tirukkural 3. Eighteen Major (d) None of the above Works of Sangam Literature 88. With reference to Mansabdari system, which D. Silappadikaram 4. Tamil text deals of the following pair is correctly matched? with philosophy and wise maxims 1. Mansabdar – Person holding rank below 500 zat Select the correct answer using the code given below: 2. Umda-i-azam – Person holding rank from 500 to below 2500 zat A B C D 3. Amir – Person holding rank of 2500 zat (a) 2 3 4 1 and above (b) 3 4 1 2 Select the correct answer using the code given (c) 4 3 2 1 below: (d) 3 2 1 4 (a) 1 only (b) 2 only 86. Match the following literatures written during Vijayanagar period with their respective (c) 3 only Authors: (d) None of the above List I List II A. Jambavati Kalyanam 1. Pingali 89. Consider the following statements regarding Suranna the Rigveda: B. Manucharitam 2. Tenali 1. Indus river is called naditama or the best Ramakrishna of the rivers in the Rig Veda. C. Panduranga 3. Allasani 2. Rigveda contains only archaic Sanskrit Mahamatyam words. Peddanna Which of the above statements is/are correct? D. Garuda Puranam 4. Krishna Deva Raya (a) 1 only Select the correct answer using the code given (b) 2 only below: (c) Both 1 and 2 A B C GSD SCORE(d) Neither 1 nor 2 (a) 1 2 3 4 (b) 2 4 1 3 90. Consider the following statements regarding the village assemblies and their administration (c) 3 1 2 4 in Chola Empire: (d) 4 3 2 1 1. Mahasabha was a gathering of the adult men in the Brahman villages called as 87. Harsha is called the last great Hindu emperor Agraharams. of India. In this context, consider the following statements: 2. The affairs of the village were managed by an executive committee to which 1. He was a staunch Hindu Ruler. educated persons owning property were 2. Kashmir, Rajasthan, , UP, Bihar, elected either by drawing lots or by and Orissa were under his direct control. rotation.

15 PTS2020/PPP-21/122019/26

Website ➡ https://upscpdf.com https://t.me/UPSC_PDF Website ➡ https://upscpdf.com https://t.me/UPSC_PDF

Which of the above statements is/are correct? 94. Consider the following statements with respect to Vijayanagar king Krishna Deva Raya: (a) 1 only 1. He took the title of “Yavanarajya (b) 2 only sthapanacharya” after defeating the (c) Both 1 and 2 Gajapati ruler Prataparudra. (d) Neither 1 nor 2 2. Amukthamalyadha is composed by him in Telugu. 91. Which of the following are the features of 3. Eight eminent scholars of Telugu known Harappan town planning? as the ‘Ashta diggajas’ were present in 1. Streets arranged in a rectangular grid his court. pattern Which of the above statements are correct? 2. Higher citadel in east and lower town in (a) 1 and 2 only west (b) 2 and 3 only 3. Use of burnt bricks for construction 4. Doors and windows opening into main (c) 1 and 3 only streets (d) 1, 2 and 3 5. Loosely covered drains alongside the main road 95. Which of the following is the correct ascending Select the correct answer using the code given order of the political structure of the Rigvedic below: times in India? (a) 1 and 2 only (a) Grama-Kula-Jana-vis-Rashtra. (b) 1, 3 and 5 only (b) Jana-Kula-vis-grama-Rashtra. (c) 2, 3, and 4 only (c) Jana-kula-grama-vis-rashtra (d) 1, 2, 3, 4 and 5 (d) Kula-grama-vis-jana-rashtra.

92. Consider the following statements regarding 96. With respect to the provincial government Pala and Pratihara Empires: during the sultanate period, arrange the 1. The directly administered territories following administrators in increasing in the Pala and Pratihara empires order of their position and role in Sultanate were divided into Bhukti, Mandala and bureaucracy: Rashtra. (a) Muqti - Shiqdar - Amil - Muqaddam 2. The Uparika collected land revenue and (b) Muqaddam - Amil - Shiqdar - Muqti maintained law and order. (c) Shiqdar - Muqti - Amil - Muqaddam Which of the above statements is/are correct? (d) Amil - Muqti - Shiqdar – Muqaddam (a) 1 only (b) 2 only 97. Consider the following statements with respect (c) Both 1 and 2 to the Gupta period: (d) Neither 1 nor 2GS SCORE1. Chandragupta I started the Gupta era in AD 319-20, which marked the date of his 93. Which of the following statements is incorrect accession. regarding trade and craft in ancient India in 2. The Gupta kingdom was enlarged the period around 200 BC to AD 250: enormously by Chandragupta I. (a) The period saw the decline of Arts and Which one of the above statements is/are crafts as compared to the pre-Mauryan incorrect? period. (a) 1 only (b) Ayatana was the guild of merchants. (b) 2only (c) The term Vyavahari was used for the one who transacts the business. (c) Both 1 and 2 (d) None of the statement is incorrect. (d) Neither 1 nor 2

PTS2020/PPP-21/122019/26 16

Website ➡ https://upscpdf.com https://t.me/UPSC_PDF Website ➡ https://upscpdf.com https://t.me/UPSC_PDF

98. Consider the following statements regarding (a) Hsuan Tsang Pala Empire: (b) Fa-hien 1. This empire was founded by Dharmapala. (c) Marco Polo 2. Dharmapala’s reign was marked by a (d) Francois Bernier tripartite struggle between the Palas, the Pratiharas and the Rashtrakutas 100. Consider the following statements: 3. This tripartite struggle was for the 1. Bairam Khan, an officer in the court of control of Kanauj and north India. Humayun was given the title of Khan-i- Which of the above statements is/are correct? Khanan. 2. The Afghan forces led by Hemu were (a) 1 and 2 only defeated at Lahore by the Mughals. (b) 3 only Which of the above statements is/are correct? (c) 2 and 3 only (a) 1 only (d) 1, 2 and 3 (b) 2 only (c) Both 1 and 2 99. Which of the following travellers visited India during the reign of Harsha? (d) Neither 1 nor 2

™™™™™

GS SCORE

17 PTS2020/PPP-21/122019/26

Website ➡ https://upscpdf.com https://t.me/UPSC_PDF Website ➡ https://upscpdf.com https://t.me/UPSC_PDF

Space for Rough Work

GS SCORE

PTS2020/PPP-21/122019/26 18

Website ➡ https://upscpdf.com https://t.me/UPSC_PDF Website ➡ https://upscpdf.com https://t.me/UPSC_PDF

Test No.

IAS Prelims - 2020 21

PTS2020/PPP-21/122019/26

GENERAL STUDIES

HISTORY - 3 Ancient + Medieval India

Answer Key

Q. 1 (c) Q. 21 (d) Q. 41 (b) Q. 61 (c) Q. 81 (b) Q. 2 (d) Q. 22 (c) Q. 42 (c) Q. 62 (a) Q. 82 (a) Q. 3 (d) Q. 23 (d) Q. 43 (c) Q. 63 (c) Q. 83 (c) Q. 4 (d) Q. 24 (b) Q. 44 (a) Q. 64 (b) Q. 84 (b) Q. 5 (c) Q. 25 (c) Q. 45 (c) Q. 65 (d) Q. 85 (a) Q. 6 (b) Q. 26 (a) Q. 46 (d) Q. 66 (b) Q. 86 (d) Q. 7 (c) Q. 27 (a) Q. 47 (b) Q. 67 (c) Q. 87 (b) Q. 8 (c) Q. 28 (d) Q. 48 (a) Q. 68 (d) Q. 88 (a) Q. 9 (b) Q. 29 (a) Q. 49 (d) Q. 69 (c) Q. 89 (d) Q. 10 (d) Q. 30 (b) Q. 50 (b) Q. 70 (d) Q. 90 (c) Q. 11 (c) Q. 31 (b) Q. 51 (a) Q. 71 (d) Q. 91 (b) Q. 12 (b) Q. 32 (b) Q. 52 (b) Q. 72 (c) Q. 92 (b) Q. 13 (b) Q. 33 (c) Q. 53 (a) Q. 73 (c) Q. 93 (a) Q. 14 (c) Q. 34 (a) Q. 54 (c) Q. 74 (b) Q. 94 (d) Q. 15 (b) Q. 35 (a) Q. 55 (c) Q. 75 (b) Q. 95 (d) Q. 16 (d) Q. 36 (d) Q. 56 (a) Q. 76 (d) Q. 96 (b) Q. 17 (d) Q. 37 (b) Q. 57 (b) Q. 77 (b) Q. 97 (b) Q. 18 (b) Q. 38 (b) Q. 58 (d) Q. 78 (a) Q. 98 (c) Q. 19 (d) Q. 39 (b) Q. 59 (d) Q. 79 (c) Q. 99 (a) Q. 20 (b) Q. 40 (a) Q. 60 (c) Q. 80 (a) Q. 100 (a)

PTS2020/PPP-21/122019/26

Website ➡ https://upscpdf.com https://t.me/UPSC_PDF Website ➡ https://upscpdf.com https://t.me/UPSC_PDF

1. Correct Option: (c) animals. The characteristic tools of the Mesolithic age are microliths or tiny Explanation: tools characterized by parallel-sided  Statement 1 is incorrect: The Middle blades taken out from prepared cores of Paleolithic tool technology is characterized such fine material as chert, chalcedony, by flake tool industry. crystal, jasper, carnelian, agate, etc. Mesolithic sites abound in Rajasthan,  Statement 2 is incorrect: Middle southern UP, central and eastern India, Paleolithic tools have mostly been found in and also south of the river Krishna. Central and Deccan India. Supplementary notes: 2. Correct Option: (d) The Stone Age Cultures Explanation:  The Old Stone Age or the Palaeolithic Culture  Statement 1 is incorrect: Despite its of India developed in the Pleistocene period aristocratic character, the nobility did of Ice Age. The Palaeolithic Age in India is provide an avenue of promotion and divided into three phases in accordance with distinction to persons drawn from the lower the type of stone tools used by the people section of society. and also according to the nature of climatic change.  Statement 2 is incorrect: Of the Hindu nobles, the Marathas formed more than half  The first phase is called Early or during Aurangzeb’s reign. Lower Palaeolithic. The main tool types in this phase were hand axes Supplementary notes: and cleavers, along with chopper- chopping tools. Lower Palaeolithic Nobility under the Mughals tools have been found over a large area,  Socially and economically, the Mughal virtually from all over India, except nobility formed a privileged class. the plains of the Indus, Saraswati, Theoretically Mughal nobility were open Brahmaputra and Ganga where raw to everyone. In practice, persons belonging material in the form of stone is not to aristocratic families, whether they available. were Indians or foreigners, had a decided  The second phase is called Middle advantage. Palaeolithic. The Middle Palaeolithic  The bulk of the Mughal nobles were drawn tool technology is characterized from the homeland of the Mughals – Turan basically by the flake tool industry. and from its neighboring areas, Tajikistan, The tools are made on flakes obtained Iran, etc. in the beginning. by striking them out from pebbles  or cobbles. The tools show regional Babur tried to win the leading Afghan variations both in as shapes and sizes. nobles to his side but the tussle between Middle Palaeolithic tools have the Mughals and the Afghans continued in mostly been found in Central India, Bihar and Bengal even under Akbar. But Deccan, Rajasthan, Maharashtra, from the times of Jahangir, more Afghans Tamil Nadu, Karnataka and Orissa. began to be recruited in the nobility.  The third phase is called Upper  From the time of Akbar, Hindus also began Palaeolithic. The basic technological to be inducted into the nobility on a regular innovation GSof the Upper Palaeolithic SCORE basis. The largest section among them period is the method of producing parallel consisted of the . sided blades from a carefully prepared  The Rajputs who were recruited to the nobility core. The upper Palaeolithic tools have were either hereditary rajas or belonged to been found in Rajasthan, parts of the aristocratic families related to or allied to the Ganga and Belan valleys, Central and raja. Thus, their incorporation into the Western India, Gujarat, nobility strengthened its aristocratic and Karnataka. character. Despite this, the nobility did  In 9000 BC began an intermediate stage provide an avenue of promotion and in Stone-Age culture, which is called distinction to persons drawn from the the Mesolithic age or Late Stone Age. It lower section of society. Thus, many intervened as a transitional phase between kayasthas and khatris were employed the Palaeolithic and the Neolithic or New at various levels in the central and Stone ages. The Mesolithic people lived provincial governments. A few of on hunting, fishing, and food gathering; them were elevated to the position of at a later stage they also domesticated a noble.

PTS2020/PPP-21/122019/26 2

Website ➡ https://upscpdf.com https://t.me/UPSC_PDF Website ➡ https://upscpdf.com https://t.me/UPSC_PDF

 Under Jahangir and Shah Jahan, the bulk the name Ashoka. The title devanampiya of the nobles already consisted of those who or ‘dear to gods’ adopted by Ashoka had been born in India. Simultaneously, was not unique but also adopted by his the proportion of Afghans, Indian Muslims ancestors. However, piyadasi or ‘good (Hindustanis), and Hindus in the nobility looking’ seems to have been his unique continued to rise. A new section among the title. Hindus which entered the nobility during  Ashokan inscriptions have been found in the period were the Marathas. Jahangir India, Nepal, Pakistan, and Afghanistan. was the first monarch who realized that Altogether, they appear at forty-seven the Marathas were ‘the hub of affairs in places, and the total number of versions the Deccan, and tried to win them over is 182 including two edicts which are to his side. The policy was continued by considered spurious. Shah Jahan. Of the Hindu nobles, the Marathas formed more than half during  It is significant that Ashokan inscriptions Aurangzeb’s reign. which were generally located on ancient  highways, have been found at six places The Mughal nobility had a number of in Afghanistan. Composed in Prakrit, they unusual features. Though often divided on were written in Brahmi script in the greater ethnic lines, it formed a composite ruling part of the subcontinent. However, in the class representing different regions and north-western part of the subcontinent religions. they appeared in Aramaic language and  It also tried to promote a composite culture by Kharoshthi script, and in Afghanistan they extending patronage to painters, musicians, were written in both Aramaic and Greek poets, both of Persian and Hindavi, and to scripts and languages. scholars.  He was the first Indian king to speak directly  Though essentially feudal in character, to the people through his inscriptions which since “land was its main source of income, carry royal orders. The inscriptions throw it had developed many characteristics of light on Ashoka’s career, his external and a bureaucracy. It was also becoming more domestic policies, and the extent of his commerce and money minded. empire.

3. Correct Option: (d) 4. Correct Option: (d) Explanation: Explanation:  Statement 1 is incorrect: Ashoka’s name • All statements are correct does not occur only in those inscription Supplementary notes: found in north or north-west India.  Statement 2 is incorrect: The title Patterns of Village Life under Mughals devanampiya or ‘dear to gods’ adopted by  It is difficult to compute the average size of Ashoka was not unique but also adopted by the holding of the peasant. The information his ancestors. available to us shows that there was a great deal of inequality in the villages. Supplementary notes:  The peasant who did not have his own Ashokan Inscriptions ploughs and bullocks of ten tilled the land  The history of Ashoka is reconstructed on the of the zamindars or the upper castes, and basis of his inscriptions which is thirty nine could eke out a bare existence. in number. TheseGS inscription are classified SCORE  The landless peasants and laborers into Major Rock Edicts, Minor Rock Edicts, often belonged to the class of people Separate Rock Edicts, Major Pillar Edicts, called ‘untouchables’ or kamin. and Minor Pillar Edicts.  Whenever there was a famine and famines  The name Ashoka occurs in copies were frequent-it was this class of peasants of Minor Rock Edict I found at three and the village artisans who suffered the places in Karnataka and at one in MP. most. Thus, altogether, the name Ashoka  occurs four times. The peasants who owned the land they tilled were called khudkasht. They paid  It is significant that Ashoka’s name does land revenue at customary rates. Some not occur in any of his inscriptions of them had many ploughs and bullocks from north or north-west India. which they let out to their poorer  The inscriptions which do not carry brethren, the tenants or muzarain who his name mention only devanampiya generally paid land revenue at a higher piyadasi, dear to the gods, and leave out rate.

3 PTS2020/PPP-21/122019/26

Website ➡ https://upscpdf.com https://t.me/UPSC_PDF Website ➡ https://upscpdf.com https://t.me/UPSC_PDF

 These two groups were the largest section Supplementary notes: among the cultivators in the village. Thus, the village society was highly unequal. Vijaynagar Empire  The Vijayanagar kingdom constituted four dynasties — Sangama (c.1336–1485 5. Correct Option: (c) CE), Saluva (c.1485–1503 CE), Tuluva Explanation: (c.1503–1570 CE) and Aravidu (till the end of the 17th century)  Both statements are correct  The foundation of Vijaynagar Empire in Supplementary notes: India was laid by Harihara and Bukka, the founders of Sangama dynasty. Rise of States in the Northeast Region  These were originally feudatories of the  The area situated between the Ganges and Kakatiyas of Warangal and after their fall, the Brahmaputra now covering Bangladesh they served as ministers in the Kampili emerged as a settled and fairly Sanskrit- state. educated region in the fifth and sixth  centuries. These two brothers founded a new city of Vidyanagar or (meaning  By AD 600 the area came to be known city of victory) on the south bank of river as Gauda and functioned as an Tungabhadra. independent state ruled by Shashanka,  The decline of the Hoysala kingdom enabled Harsha’s adversary. Harihara and Bukka to expand their newly  The deltaic portion of Bengal formed by founded kingdom. By c. 1346 CE, they the Brahmaputra and called Samatata brought the whole of the Hoysala kingdom was made to acknowledge the authority of under their control. Samudragupta in the fourth century.  Bukka succeeded his brother in c.1356 and  By the seventh century, there existed the ruled till 1377 CE. During his reign, the state of the Khadgas in the Dhaka area, Sultanate of Madurai was eliminated in the literally swordsmen. year 1377 and the Vijaynagara kingdom comprised the whole of south India up to  There was also the kingdom of a brahmana Rameshwaram, including the Tamil country feudatory called Lokanatha and that of the as well as that of the Cheras (Kerala). Ratas, both in the Comilla area.  All these princes of south-east and central 7. Correct Option: (c) Bengal issued land grants in the sixth and seventh centuries. Explanation:  The practice of land charters attested to  Option (c) is correct the expansion of cultivation and rural Vedas Characteristic settlements. A. Rigveda 2. Collection of hymns  A fiscal and administrative unit called Dandabhukti was formed in the border B. Samveda 4. Collection of songs areas between Bengal and Orissa. C. Yajurveda 1. Collection of sacrificial  Danda means punishment and bhukti formulae enjoyment. The GSunit was formed apparently SCORE D. Atharvaveda 3. Collection of spells and to pacify and suppress the tribal inhabitants charms of that region and may have promoted Sanskrit and other elements of culture in Supplementary notes: the tribal areas. The Vedas  This was also true of Vardhamanabhukti  Vedas are a collection of hymns, prayers, (Burdwan) of which we hear in the sixth charms, litanies and sacrificial formulae. century.  The Vedas formed the earliest segment of Vedic literature and amongst the Vedas, 6. Correct Option: (b) Rigveda is the oldest. Explanation:  There are four Vedas, namely:   Option (b) is correct: The correct sequence Rigveda - a collection of hymns is Sangama dynasty - Saluva dynasty -  Samveda - a collection of songs Tuluva dynasty - Aravidu dynasty. mostly taken from Rig Veda

PTS2020/PPP-21/122019/26 4

Website ➡ https://upscpdf.com https://t.me/UPSC_PDF Website ➡ https://upscpdf.com https://t.me/UPSC_PDF

 Yajurveda - a collection of sacrificial  Babur was deeply learned in Persian and formulae Arabic, and is regarded as one of the two most famous writers in the Turkish language  Atharvaveda - a collection of spells and which was his mother tongue.His famous charms memoirs, the Tuzuk-i-Baburi, is considered one of the classics of literature. His other 8. Correct Option: (c) works include a masnavi and the Turkish translation of a well-known Sufi work. He Explanation: was a keen naturalist, and has described  Statement 2 is incorrect: Arches and the flora and fauna of India in considerable Domes were introduced by Turks during the detail. He laid out a number of formal Delhi Sultanate. gardens with running water thereby establishing a tradition of building  Statement 3 is incorrect: Gunpowder gardens. was already known in India before Babur’s arrival. His victories led to rapid  Babur introduced a new concept of the state popularization of gunpowder and artillery which was to be based on the strength and in India. prestige of the Crown, absence of religious and sectarian bigotry, and the careful Supplementary notes: fostering of culture and the fine arts. Significance of Babur’s Advent into India 9. Correct Option: (b)  Babur’s advent into India was significant Explanation: from many points of view.  Option (b) is correct:  For the first time since the downfall of the Kushana Empire, Kabul and Important Archeological Findings Qandhar became integral parts of Harappan an empire comprising north India. Sites Since these areas had always acted as A. Harappa 2. Red sandstone male torso staging places for an invasion of India by dominating them Babur and his B. Banawali 3. City with radial streets successors were able to give to India C. Ropar 4. Dog buried with human security from external invasions for almost 200 years. D. Surkotada 1. Remains of horse bones  Economically, the control of Kabul and Supplementary notes: Qandhar strengthened India’s foreign trade since these two towns were the starting Important sites of Indus Valley points for caravans meant for China in the Civilization east, and the Mediterranean seaports in the  Some of the important sites of the Indus west. Thus, India could take a greater share Valley civilization and their archaeological in the great trans-Asian trade. findings are:  Babur introduced a new mode of warfare in  Harappa: Two rows of six granaries with India. Although gunpowder was known big platform, stone symbol of lingam and in India earlier, Babur showed what yoni, mother goddess figure, wheat and a skilled combination of artillery and barley in wooden mortar, dice, copper cavalry could GSachieve. His victories SCORE led scale and mirror, a sculpture of dog to rapid popularization of gunpowder chasing a deer in bronze metal, and a and artillery in India. red sand stone male torso have been  By his new military methods as well as by excavated. his personal conduct, Babur re-established  Mohenjodaro: The citadel, the great the prestige of the Crown which had been bath, the great granary, post cremation eroded since the death of Firoz Shah burial, sculpture of bearded priest, the Tughlaq. Although Sikandra Lodi and famous bronze statue of the Dancing Ibrahim Lodi had tried to reestablish the Girl and Pashupati seal. prestige of the Crown, Afghan ideas of tribal independence and equality had resulted  (Gujarat): Giant water only in a partial success. Babur had the reservoir, unique water harnessing prestige of being a descendant of two of the system, stadium, dams and most famous warriors of Asia, Changez and embankments, inscription comprising 10 Timur. None of his nobles could, therefore, large sized signs like an advertisement claim a status of equality with him. board

5 PTS2020/PPP-21/122019/26

Website ➡ https://upscpdf.com https://t.me/UPSC_PDF Website ➡ https://upscpdf.com https://t.me/UPSC_PDF

 (Gujarat): Important site for  He was the only Delhi Sultan who had naval trade had a Dockyard, practice of received a comprehensive literary, religious, burial of cremated remains, rice husk, and philosophical education and is known fire altars, painted jar, modern day to have built the fort of Adilabad and chess, terracotta figure of horse and the city of Jahanpanah. ship, instruments for measuring 45, 90 and 180 degree angles, practice of burial of cremated remains. 11. Correct Option: (c)  Rakhigarhi (Haryana) is considered Explanation: to be the largest site of Indus Valley  Statement 1 is incorrect: The samaharta civilization. Granary, cemetery, drains, was the highest officer in charge of terracotta bricks have been found assessment and collection.  Ropar (Punjab): Dog buried with  Statement 2 is incorrect: The sannidhata human oval pit burials, copper axe. was the chief custodian of the state treasury and storehouse.  Surkotada (Gujarat): First actual remains of the horse bones. Supplementary notes:  Banawali (Haryana): Toy plough, Taxation System during Mauryan Age barley grains, lapis lazuli, fire altars  oval shaped settlement, only city with The Maurya period constitutes a landmark radial streets. in the system of taxation in ancient India. Kautilya names many taxes which were  Alamgirpur (Uttar Pradesh): Eastern collected from peasants, artisans, and most site of IVC. Major findings are traders. broken blade made of copper, ceramic  This required a strong and efficient items and Impression of a cloth on a machinery for assessment, collection, and trough. storage. The Mauryas attached greater  Balathal and Kalibangan importance to assessment than to storage (Rajasthan): Bangle factory, toy carts, and deposit. bones of camel, decorated bricks, citadel  The samaharta was the highest officer and lower town, fire altar. in charge of assessment and collection, and the sannidhata was the chief 10. Correct Option: (d) custodian of the state treasury and storehouse. The assessor-cum-collector Explanation: was far more important than the chief treasurer.  All statements are correct  The damage inflicted on the state by the Supplementary notes: first was thought to be more serious than Mohammad Bin Tughlaq any inflicted by the second.   Mohammad Bin Tughlaq is considered an In fact, elaborate machinery for assessment ill-fated idealist owing to his ambitious was first set up during the Maurya period. The list of taxes mentioned in the schemes and novel experiments. Arthashastra is impressive, and, if these  He transferred his capital from Delhi to were really collected, very little would have Devagiri (Daulatabad) but the capital been left to the people to live on. shifted back toGS Delhi after two yearsSCORE because of the lack of water supply in Daulatabad. 12. Correct Option: (b)  He modeled the idea of token currency on Explanation: the basis of the Chinese ruler Kublai Khan  Statement 2 is incorrect: It was the mir who issued paper money in China. bakhshi and not the diwan/wazir, who was  He issued copper coins at par with the considered the head of the nobility. value of the silver tanka coins. Later Supplementary notes: on, he repealed his verdict, and all coins were redeemed in silver/gold, making the Organization of Central and Provincial treasury empty. Governments  During his reign, the famous traveler  Akbar paid great attention to the organization Ibn Batuta came to India in 1334 CE of the central and provincial governments. and acted as a Qazi at Delhi for eight His system of central government was years. based on the structure of government which

PTS2020/PPP-21/122019/26 6

Website ➡ https://upscpdf.com https://t.me/UPSC_PDF Website ➡ https://upscpdf.com https://t.me/UPSC_PDF

had evolved under the Delhi Sultanate, but  Statement 3 is incorrect: Vidatha the functions of the various departments disappeared from the later vedic texts. were carefully reorganized and meticulous rules and regulations were laid down for the Supplementary notes: conduct of affairs. Later Vedic polity  Akbar reorganized the central  It covers the time period around 1000-600 machinery of administration on the BC. basis of the division of power between  various departments and of checks and There was a conspicuous urge in the later balances. Vedic texts for increasing the power of the raja through performances of elaborated  The head of the revenue department was the sacrifices such as Vajpeya,asvamedha, etc. Wazir. Under Akbar, generally the wazir did  not hold a high mansab. Many nobles held The raja’s position became more powerful mansabs which were higher than his. Thus, than his Rigvedic counterpart. he was no longer the principal adviser to the  The polity gave greater stress on ruler, but one who was an expert in revenue hereditary rulership, paving the way affairs. Thus, in practice, the concept of for dynastic succession as an integral an all-powerful wazir had been given up. feature of a monarchical polity of To emphasize this point, Akbar generally subsequent times. used the title of diwan or diwan-i-ala in  preference to the word wazir. Despite the growth of the ruler’s power, the later Vedic period did not experience  The head of the military department was a monarchical system. It was a proto- called the mir bakhshi. It was the mir state, on the threshold of a state bakhshi and not the diwan who was system. considered the head of the nobility.  Recommendations for appointment to The absence of a regular well-defined mansabs or for promotions etc. were made revenue as the rate of Bali was nowhere to the emperor through the mir bakhshi. specified indicating the absence of its proper and adequate assessment resulted in the  The mir bakhshi was also the head of the absence of enough resources which precluded intelligence and information agencies of the the formation of military organization. empire. Intelligence officers (barids) and  Tribal units were mustered in times of news reporters (waqia-navis) were posted war and, according to one ritual, for success to all parts of the empire. Their reports in war, the king had to eat along with his were presented to the emperor at the court people (vis) from the same plate. through the mir bakhshi.  In later Vedic times, the Rig Vedic tribal  The third important officer was the mir assemblies lost importance, and royal saman He was in charge of the imperial power increased at their cost. The vidatha household, including the supply of all completely disappeared. Sabha and the provisions and articles for the use of Samiti appeared more prominently in the inmates of the haram or the female later Vedic texts. apartments.  The sabha was a smaller select body and  The fourth important department was the also functioned as the lower court, while the judicial department headed by the chief qazi. Samiti was the larger GeneralAssembly of This post was sometimes combined with that the people. Accordingly, the latter is referred of the chief sadr who was responsible for all to as expressing the voice of vis (people). charitable and religiousGS endowments. SCORE  Akbar divided the empire into twelve subas in 1580. A governor (subedar), a diwan, a 14. Correct Option: (c) bakhshi, a sadr, a qazi, and a waqia-navis Explanation: were appointed to each of the provinces. Thus, orderly government based on the  Both statements are correct principle of checks and balances was Supplementary notes: extended to the provinces. Lodhi Dynasty 13. Correct Option: (b)  Bahlol Lodhi laid the foundation of the Lodhi dynasty in India in 1451 after Explanation: defeating Alauddin Alam Shah of the  Statement 1 is incorrect: Later Vedic Sayyid dynasty. polity did not experience a monarchical  He considered himself one of the Afghan system. peers and not the king.

7 PTS2020/PPP-21/122019/26

Website ➡ https://upscpdf.com https://t.me/UPSC_PDF Website ➡ https://upscpdf.com https://t.me/UPSC_PDF

 He was succeeded by Sikander Lodhi whose  Animals are the most frequently depicted real name was Nizam Khan. subject either alone or in large and small  groups and shown in various poses. The Sikander Lodhi was a good administrator. animals are drawn in bold outline, He laid many roads and provided many and the bodies are sometimes filled irrigational facilities for the benefit of the in completely or partially with peasantry. crosshatching.  Sikander Lodhi introduced a new  Depiction of human figures in rock measurement yardstick, the Gazz-i- paintings is quite common. These are Sikandar, and a system of auditing of in simple outline forms as well as with accounts. However, he showed little hatched body. The humans are shown in tolerance towards the non-Muslims and re- various activities, such as dancing, running, imposed jizya on them. hunting, playing games and engaged in battle.  Sikander Lodhi founded Agra in c.1504 CE and wrote Persian verses under the  One of the most common scenes name ‘Gulrakhi’. depicted in these paintings is of group hunting and several paintings depict  Ibrahim Lodhi was the eldest son of Sikander grazing activity and riding scenes. Lodhi. Babur marched against Delhi and defeated and killed Ibrahim Lodhi in the  The colours used in making these drawings First Battle of Panipat (c.1526 CE). are deep red, green, white and, yellow  The Sultanate of Delhi, which had its birth on the battlefield of Tarain in c. 1192 CE, 16. Correct Option: (d) breathed its last in c.1526 CE, a few miles Explanation: away on the battlefield of Panipat  All statements are correct 15. Correct Option: (b) Supplementary notes: Explanation: State and Religion under Akbar   Statement 3 is incorrect: The animals are Akbar followed a policy of broad religious drawn in bold outline, and the bodies are toleration. sometimes filled in completely or partially  In 1564, he abolished the Jizyah which with crosshatching. Human figures in was sometimes used by the Ulama to simple outline forms as well as with hatched humiliate non-Muslims and was often body. considered a symbol of Muslim domination and superiority. He had earlier abolished Supplementary notes: the pilgrim tax, and the practice of forcible Prehistoric Rock Art conversion of prisoners of war.   The people of the Palaeolithic and Mesolithic The liberal principles of the empire were ages practiced painting. The pre-historic strengthened by bringing able Hindus into paintings were generally executed on the nobility. rocks and these rock engravings were  Akbar’s attitude towards his Hindu called Petroglyphs. subjects is closely linked with his views of how a sovereign should behave  Almost all the rock-shelters in India towards his subjects. According to occupied by the Upper Palaeolithic and Abul FazI, the office of a true ruler was Mesolithic people,GS and many others asSCORE well a very responsible one which depended contain rock-paintings depicting a variety of on divine illumination (farr-i-izadi). subjects, chiefly animals, or scenes including Hence, no one could stand between both people and animals. God and a true ruler. A true ruler  Some of the important rock-painting sites was distinguished by a paternal love are Murhana Pahar in Uttar Pradesh, towards his subjects without distinction Bhimbetka, Adamgarh, and Lakha Juar of sect or creed and it was his duty to in Madhya Pradesh and Kupagallu in maintain equilibrium in society by not Karnataka. Bhimbetka in MP is a allowing the dust of sectarian strife to striking site having over 500 painted rise. All this constituted what has been rock shelters distributed in an area called the policy of sulh-i-kul or ‘peace to all’. of 10 sq. km. and the rock paintings extend from the Upper Palaeolithic to  Akbar was deeply interested in religion the Mesolithic age and in some series and philosophy. In 1575, Akbar built a hall even up to recent times. called Ibadat Khana or the Hall of Prayer

PTS2020/PPP-21/122019/26 8

Website ➡ https://upscpdf.com https://t.me/UPSC_PDF Website ➡ https://upscpdf.com https://t.me/UPSC_PDF

at his new capital, . To this Financial Crisis he called selected theologians, mystics and  The enormous expenditure on the those of his courtiers and nobles who were army and payment to the bureaucracy known for their scholarship and intellectual created a financial crisis for the Maurya attainments. The proceedings, at first, empire. It seems that Ashoka made large were confined to the Muslims and were donations to the Buddhist monks which left later opened to people of all religions- the royal treasury empty. Towards the end, Christians, Zoroastrians, Hindus, in order to meet expenses, they were obliged Jains, even atheists. to melt gold images.  But Akbar was less successful in his effort to Oppressive Rule find a meeting ground between the votaries of different religions in the country. The  Oppressive rule in the provinces was an debates in the Ibadat Khana had not led to important cause of the break-up of the a better understanding between different empire. In the reign of Bindusara, the religions, but to greater bitterness, as the citizens of Taxila bitterly complained representatives of each religion denounced against the misrule of wicked the others and tried to prove that their bureaucrats (dushtamatyas). religion was superior to others. Hence, in New Knowledge in the Outlying Areas 1582, Akbar discontinued the debates in the Ibadat Khana.  The regular use of iron tools and weapons in the peripheral provinces coincided with the  Badayuni asserts that as a result, decline and fall of the Maurya empire. On Akbar gradually turned away from the basis of the material culture acquired Islam and set up a new religion which from Magadha, new kingdoms could be was compounded of many existing founded and developed. religions-Hinduism, Christianity, Zoroastrianism, etc. The word used by Neglect of the North-West Frontier and Abul Fazl and Badayuni for the so called the Great Wall of China new path was tauhid-i-ilahi which  literally means ‘Divine Monotheism’ Ashoka was unable to pay attention to which was later called as Din-i-ilahi or safeguard the passes through the north- ‘Divine Faith’. western frontier .The Scythians were in a state of constant flux. Due to the use of the horse, they posed a serious danger to the 17. Correct Option: (d) settled empires in China and India. Explanation:  The Chinese ruler Shih Huang Ti (247–10 BC) constructed the Great  All statements are correct Wall of China in about 220 BC to shield Supplementary notes: his empire against the attacks of the Scythians, but Ashoka took no such Causes of the fall of the Maurya Empire measures.  The Magadhan empire, which had been  Naturally, when the Scythians made a push reared by successive wars culminating in the towards India, they forced the Parthians, conquest of Kalinga, began to disintegrate the Shakas, and the Greeks to move towards after the exit of Ashoka in 232 BC. Several this subcontinent. causes seem to have brought about the decline and fall of the Maurya empire. GS SCORE18. Correct Option: (b) Brahmanical Reaction Explanation:  The brahmanical reaction began as a result of Ashoka’s policy. He issued his edicts • Statement 2 is incorrect: Qutub-ud-din in Prakrit and not in Sanskrit. The anti- Aibak did not issue any coins during his rule. sacrifice attitude of Buddhism adopted by Supplementary notes: Ashoka adversely affected the incomes of brahmanas. Further, Ashoka appointed Slave Dynasty rajukas to govern the countryside  Invasions by and Mohd. and introduce vyavaharasamata and Ghori into India ultimately resulted in the dandasamata. This meant the same establishment of the Delhi Sultanate. civil and criminal law for all varnas. But the Dharmashastra compiled  Five different dynasties–the Slave, the by the brahmanas prescribed varna Khalji, the Tughlaq, the Sayyids, and the discrimination. Naturally this policy Lodhi’s are collectively referred to as the infuriated the brahmanas. Delhi Sultanate.

9 PTS2020/PPP-21/122019/26

Website ➡ https://upscpdf.com https://t.me/UPSC_PDF Website ➡ https://upscpdf.com https://t.me/UPSC_PDF

 Qutub-ud-din Aibak founded the Slave  Cremation developed as an Aryan dynasty (also called the Mameluk trait. Its practice is supported by the Vedic, dynasty) in India. Avestan, and Homeric texts.  He assumed the title of ‘Sultan’ and made  Animal sacrifice was an important Aryan Lahore his capital. ritual. However, given its almost universal  He did not issue any coins and was practice among pastoral tribal people, it formally recognized after three years. is difficult to make much of it. Animal sacrifice may have prevailed among many  He gave a lot of liberal donations and was tribal peoples, but the horse sacrifice famously known as ‘Lakh Baksh’ or giver of was typical of the Indo-Europeans, lakhs for his Magnanimity. particularly of the Vedic people.  The first mosque in India “Quwwat-ul-  Though two hymns are devoted to the horse Islam” in Delhi was built by him. He is sacrifice in the tenth book of the Rig Veda, also known to have built Arhai din ka the later Vedic texts transform the sacrifice jhonpara in Ajmer. into asvamedha. Animals may have  The construction of Qutub Minar was been sacrificed in pre-Vedic times in the started by him (only the first storey) in subcontinent, but despite cut marks found the memory of the famous Sufi saint, on the bones, it cannot be said that the Khwaja Qutbuddin Bakthiyar Kaki. It horses were killed for religious purposes. was later completed by Iltutmish in around 1220. 20. Correct Option: (b) 19. Correct Option: (d) Explanation:  Explanation: Statement 1 is incorrect: The first Battle of Panipat was fought between Babur and  All statements are correct Ibrahim Lodi in 1526. Supplementary notes:  Statement 2 is incorrect: The Battle of Khanwa was fought between Babur and Aryan Culture Rana Sanga in 1527.  Texts for Traits of Aryan Culture: The Supplementary notes: principal traits of Aryan culture are set out by Vedic, Iranian, and Greek literary Battle of Panipat (20 April 1526) texts and cognate terms found in the proto-  Indo-European languages. The Rig Veda is The battle of Panipat is regarded as one of assigned to roughly 1500 BC, although the the decisive battles of Indian history. It was later additions might be as late as 1000 BC. an inevitable conflict between Ibrahim The earliest parts of the Zend-Avesta are Lodi and Babur. It broke the back of Lodi roughly attributed to 1400 BC. power, and brought under Babur’s control the entire area upto Delhi and Agra. The  The Horse, its Domestication and Diffusion: treasures stored up by Ibrahim Lodi at The horse is regarded as an indispensable Agra relieved Babur from his financial trait of the Aryan culture, for it plays a difficulties. crucial role in the life of the early Indo- Europeans. In its various forms, the term  Babur had to wage another two hard-fought asva (horse) occurs 215 times in the Rig battles, one against Rana Sanga of Mewar; Veda; no otherGS animal is mentioned SCORE so and-the other against the eastern Afghans, frequently. The Indo-Europeans widely before he could consolidate his hold on this used horse-drawn chariots. Aryans area. succeeded everywhere because they  possessed chariot driven by horses & Viewed from this angle, the battle of Panipat possibly with coats of mail & better armory. was not as decisive in the political field as They also introduced these things for the has been made out: Its real importance lies 1st time in west Asia & India. in the fact that it opened a new phase in the struggle for domination in north India.  The pit-dwelling can also be associated with the Aryan culture, and may Battle of Khanwa (1527) have originated in cold conditions.  The establishment of an empire in the The practice of pit-dwelling prevailed in Indo Gangetic valley by Babur was a Burzahom near Srinagar in Kashmir and threat to Rana Sanga. Sanga set afoot also in Haryana. This may be due to the Central Asian influence on the borders of preparations to expel Babur, at any rate, to Kashmir. confine him to the Punjab.

PTS2020/PPP-21/122019/26 10

Website ➡ https://upscpdf.com https://t.me/UPSC_PDF Website ➡ https://upscpdf.com https://t.me/UPSC_PDF

 Babur accused Rana Sanga of breach of  It was during Chandragupta’s reign agreement as he promised to join him against that the Chinese pilgrim Fa-hsien Ibrahim Lodi, but made no move while he (AD 399–414) visited India and wrote (Babur) conquered Delhi and Agra. an elaborate account of the life of its people.  Having carefully selected a site, Babur entrenched himself at Khanwa about 40 km from Agra. The Battle of Khanwa was 22. Correct Option: (c) fiercely contested and resulted in defeat of Sanga’s forces. Explanation:  The battle of Khanwa secured Babur’s • Option (c) is correct: The correct position in the Delhi-Agra region and chronological order is: thus was more decisive than the Battle 1. Bahram Shah at Panipat. Babur strengthened his position further by conquering the chain of 4. Alauddin Masud Shah forts-Gwalior, Dholpur, etc., east of Agra. 2. Nasiruddin Mahmud He also annexed large’ parts of Alwar from 3. Balban Hasan Khan Mewati. Supplementary notes: 21. Correct Option: (d) Slave dynasty rulers Explanation:  The fall of Raziya paved way for Bahram Shah for the ascendancy with the support of  Option (d) is incorrect: Vikrama samvat noble elites, The Forty. or era in 58– 57 BC was started by Ujjain  ruler Shakari. However he was later murdered by his own army and the chiefs chose Alauddin Masud Supplementary notes: Shah, Son of Ruknuddin Firuz Shah and nephew of Raziya Sultan to become the next Chandragupta II ruler.  The Gupta Empire saw its peak during  He became infamous for his fondness for the reign of Chandragupta II (AD 380– entertainment and wine. 412).  By the year 1246, the chiefs became upset  He extended the limits of the empire by with his increasing hunger for more power marriage alliance and conquest. in the government and replaced him with  Chandragupta married his daughter Nasiruddin Mahmud, the grandson of Prabhavati to a Vakataka prince of the Iltutmish. brahmana caste and ruled in central India  He had ascended to the throne with the and exercised indirect control over the aid of Ulugh Khan/Balban (member of the Vakataka kingdom in central India. Chahalgani), who himself assumed the  He conquered Mathura from the Kushans position of Naib (deputy or regent). and occupied western Malwa and Gujarat,  Due to internal conflicts, as per historians which had for about four centuries been such as Ibn Battuta and Isami, Balban under the rule of the Shaka Kshatrapas. poisoned his master Nasiruddin and ascended the throne in the year 1266.  This gave him control over the western sea coast, famous forGS trade and commerce. SCORE  The chief city Ujjain of Malwa region was 23. Correct Option: (d) made the second capital by Chandragupta Explanation: II.  Option (d) is correct  He adopted the title of Vikramaditya, which had been first used by an Ujjain ruler in Supplementary notes: 58–57 BC as a mark of victory over the Pulakeshin II Shaka Kshatrapas of western India. This Ujjain ruler is traditionally called Shakari  The first important event in long conflict or the enemy of the Shakas. The Vikrama between the Pallavas and the Chalukyas samvat or era was started in 58– 57 BC took place during the reign of Pulakeshin II by Shakari. (AD 609–42), the most famous Chalukya king.  The court of Chandragupta II at Ujjain was adorned by numerous scholars  He subjugated the Kadamba capital at including Kalidasa and Amarasimha. Banavasi and compelled the Gangas of

11 PTS2020/PPP-21/122019/26

Website ➡ https://upscpdf.com https://t.me/UPSC_PDF Website ➡ https://upscpdf.com https://t.me/UPSC_PDF

Mysore to acknowledge his suzerainty. intervention, Bahadur Shah patched up a He also defeated Harsha’s army on the treaty with the rana, leaving the fort in his Narmada and checked his advance towards hands after extracting a large indemnity in the Deccan. cash and kind.  In his conflict with the Pallavas, he almost  During the next year and a half, reached the Pallava capital, but the Pallavas Humayun spent his time in building purchased peace by ceding their northern a new city at Delhi, which he named provinces to Pulakeshin II. Around AD Dinpanah. The building of Dinpanah was 610 Pulakeshin II also conquered the meant to impress friends and foes alike. It entire area between the Krishna and could also serve as a second capital in case the Godavari, which came to be known Agra was threatened by Bahadur Shah. as the province of Vengi.  Here, a branch of the main dynasty was set 25. Correct Option: (c) up and is known as the eastern Chalukyas of Vengi. However, Pulakeshin’s second Explanation: invasion of Pallava territory ended in  Both statements are correct failure. The Pallava king Narasimhavarman (AD 630–68) occupied the Chalukya capital Supplementary notes: at Vatapi in about AD 642, when Pulakesin Early Vedic Period II was probably killed in a battle against the Pallavas. Narasimhavarman assumed  Cattle Rearing and Agriculture: the title of Vatapikonda or the conqueror of  Agriculture was well known to pre Vatapi. He is also said to have defeated the Aryans & ploughshare is 1st mentioned Cholas, the Cheras, the Pandyas, and the in early part of Rig Veda but agriculture Kalabhras. was perhaps used to produce fodder only. 24. Correct Option: (b)  However, there are so many references to the cow and the bull in the Rig Veda that Explanation: the Rig Vedic people can be called a  Option (b) is correct predominantly pastoral people.  Supplementary notes: Most of the wars were fought for the sake of cows as the term for war Humayun in Rig Veda is Gavishthi i.e. search for cows (Hence cows seems to have  Humayun succeeded Babur in December been most important wealth). 1530 at the young age of 23. He had to grapple with a number of problems left  The Rig Vedic people may have behind by Babur. The administration had occasionally occupied pieces of land for not yet been consolidated, and the finances grazing, cultivation, and settlement, but were precarious. The Afghans had not been land did not form a well-established type subdued, and were nursing the hope of of private property. expelling the Mughals from India. Finally,  Social Differentiation: there was the Timurid legacy of partitioning the empire among all the brothers.  The factor that contributed most to  the creation of social divisions was the Humayun also had to deal with the rapid conquest of the indigenous inhabitants growth of power of the Afghans in the east by the Indo-Aryans. and Bahadur Shah,GS the ruler of Gujarat SCORE in the west.  The dasas and the dasyus, who were conquered by the Aryans, were  In 1532, at a place called Dadrah, he defeated treated as slaves and shudras. The the Afghan forces which had conquered Rig Veda mentions the arya Varna Bihar and overrun Jaunpur in eastern and dasa Varna (Varna was the term Uttar Pradesh. After this success, Humayun used for colour). besieged Chunar which had recently come  in the possession of an Afghan sardar, Unequal distribution of the spoils of Sher Khan. Sher Khan however, persuaded war created social inequalities, and this Humayun to allow him to retain possession aided the rise of princes and priests at of the fort and in return, he promised to be the cost of the common tribal people. loyal to the Mughals, and sent one of his  However as the economy was mainly sons to Humayun as a hostage. pastoral and not food producing, the  After this Humayun moved from Agra scope for collecting regular tributes from to Gwalior, and due to fear of Mughal the people was very limited.

PTS2020/PPP-21/122019/26 12

Website ➡ https://upscpdf.com https://t.me/UPSC_PDF Website ➡ https://upscpdf.com https://t.me/UPSC_PDF

 The tribal elements in society were 27. Correct Option: (a) stronger and social divisions based on the collection of taxes or accumulation Explanation: of landed property did not exist, and  Statement 1 is incorrect: The Harappan thus the society was still tribal and Civilization marked the beginning of first egalitarian. urbanization in India.  Statement 2 is incorrect: Harappans 26. Correct Option: (a) probably used wooden ploughs drawn by oxen or camel. Explanation: List-I List-II Supplementary notes: Harappan Civilization A. Jalauddin Khalji 2. Settled New  Agriculture Musalmans near  Delhi Agriculture was generally practiced along the river banks most of which B. Alauddin Khalji 4. Renamed Chittor to were flooded during the summer and Khizrabad monsoons. C. Qutbuddin 1. Deposed Malik  The granaries at some Harappan Mubarak Shah Kafur cities clearly suggest that cereals were produced in such large quantities that D. Nasiruddin 3. Only Hindu convert not only were all the immediate needs Khusrau to sit on the throne of people duly met with, but there was of Delhi also enough reserve to face any future emergency. Supplementary notes:  The principal cereals seem to have been Khalji Dynasty wheat and barley. Rice, though known,  Jalaluddin is known to have followed the was a favored grain. Six varieties of policy of tolerance. millets including ragi, kodon, sanwa, and jowar were cultivated, as also peas  He repelled the attack of the Mongols and beans. on Delhi Sultanate and allowed the  No hoe or ploughshare has been converted Mongols to settle near Delhi discovered, but the furrows who were called the ‘New Musalmans’. discovered in the pre-Harappan  Alauddin Khalji reversed Jalaluddin’s phase at Kalibangan indicate policy of tolerance and pursued ruthless that the fields were ploughed in governance. Rajasthan during the Harappan period. The Harappans probably  He made an expedition against Gujarat, used the wooden plough drawn by Rajputana, and the Deccan states and is oxen, and camels may also have known to have renamed Chittor to been used for this purpose. Khizrabad after his son Khizr Khan.  Gabarbands or nalas enclosed by dams  Malik Kafur was the favorite of Alauddin for storing water were a feature in parts and he raised a minor son of Alauddin to the of Baluchistan and Afghanistan, but throne and imprisoned or blinded his other channel or canal irrigation was probably sons, without encountering any opposition not practiced. from the nobles.GS SCORE  Domestication of animals  Qutbuddin Mubarak Shah deposed  Although the Harappans practiced of Malik Kafur to sit on the throne of agriculture, animals were raised on a Delhi after Alauddin. large scale. Oxen, buffaloes, goats, sheep,  Soon after accession to the throne, Mubarak and pigs were domesticated. Humped Shah was killed by Nasiruddin Khusrau. bulls were favored by the Harappans.  Nasiruddin Khusrau Shah was the only  Evidence of the horse comes from a Hindu convert to sit on the throne of superficial level of Mohenjodaro and Delhi. from a doubtful terracotta figurine from Lothal. The remains of a horse  In 1320, a group of officers led by Ghiyasuddin are reported from Surkotada, situated Tughlaq raised the banner of revolt which in west Gujarat, and relate to around led to open rebellion and a battle was fought 2000 BC but the identity is doubtful. In outside the capital in which Khusrau was any case, the Harappan culture was not defeated and killed. horse-centered.

13 PTS2020/PPP-21/122019/26

Website ➡ https://upscpdf.com https://t.me/UPSC_PDF Website ➡ https://upscpdf.com https://t.me/UPSC_PDF

 Pashupati seal discovered at Mohen- “Sijada” (prostration) and “Paibos” jo-daro , shows a seated figure of (kissing the Sultan’s feet) to prove his a Yogi, probably Shiva Pashupati, superiority over the nobles. surrounded by four animals - a rhino, a buffalo, an elephant and a tiger. This seal may throw light on 29. Correct Option: (a) the religion of the Harappan age. Most of these seals have a knob at the back Explanation: through which runs a hole and it is  Statement 3 is incorrect: Maurya believed that they were used by different constructions made of burnt bricks have guilds or merchants and traders for been found in Bihar and UP. stamping purposes. When not in use they could be worn round the neck or the Supplementary notes: arm like an amulet. Spread of Material Culture  Technology and Crafts  It appears that the contacts established  The rise of towns in the Indus zone by administrators, traders, and Jaina and was based on agricultural surplus, Buddhist monks led to the spread of the the making of bronze tools, various material culture of the Gangetic basin other crafts, and widespread trade to areas situated on the periphery of the and commerce. This is known as the empire. first urbanization in India, and the  Harappan urban culture belongs to The new material culture in the Gangetic the Bronze Age. basin was based on an intensive use of iron, the prevalence of writing, punch-marked  The people of Harappa used many coins, an abundance of beautiful pottery tools and implements of stone, but they called Northern Black Polished Ware, the were very well acquainted with the introduction of burnt bricks and ring wells, manufacture and use of bronze. The and above all, on the existence of towns in bronze tools and weapons recovered north-eastern India. from the  Thus, the Maurya period witnessed a rapid  The bronze tools and weapons recovered development of material culture in the from the Harappan sites contain a Gangetic plains. smaller percentage of tin.  Given the access to the rich iron ores of south Bihar, people used more and more of this 28. Correct Option: (d) metal. This period provide evidences Explanation: of socketed axes, hoes, spades, sickles, and ploughshares.  Option (d) is correct: Customs such as  “Sijada” and “Paibos” were introduced by Besides these iron implements, the spoked Balban. wheel also began to be used.  Supplementary notes: In the end of the Maurya period burnt bricks were used for the first time in north-eastern Balban Rule India. Maurya constructions made of  Balban was undoubtedly one of the burnt bricks have been found in Bihar main architects of the Delhi Sultanate, and UP. Houses were made of bricks, particularly of its form of government and and also timber which was available in GS SCOREabundance because of the dense vegetation institutions. in ancient times.  Balban’s experience as the regent made  him understand the problems of Delhi Excavations show that logs of wood were Sultanate. He knew that the real threat to also used as an important line of defence the monarchy was from the nobles called against flood and invasion the Forty.  Similarly, ring wells, which were first  He was convinced that only by enhancing constructed under the Mauryas in the the power and authority of the monarchy he Gangetic plains spread beyond the could solve this problem. heart of the empire.  He got every member of the Iltutmish family killed and gave a deadly blow to 30. Correct Option: (b) Chahalgani. Explanation:  Balban introduced rigorous court discipline and new customs such as  Option (b) is correct

PTS2020/PPP-21/122019/26 14

Website ➡ https://upscpdf.com https://t.me/UPSC_PDF Website ➡ https://upscpdf.com https://t.me/UPSC_PDF

Supplementary notes:  Next in rank to the king was the senani or the head of the army. He used spears, axes, Ibrahim Adil Shah II swords, etc.  The successor of Ali Adil Shah, Ibrahim Adil  We do not learn of any officer concerned Shah II (1580-1627), ascended the throne of with the collection of taxes. In all Sultanate of Bijapur at the age of nine. probability, the people made voluntary  He was very solicitous to the poor, and offerings called bali to the rajan. had the title of ‘abla baba’ or ‘Friend of the  Poor’. The officer who enjoyed authority over a large stretch of land or pasture  He was deeply interested in music, and ground was called vrajapati. He led composed a book called Kitab-i-Nauras in to battle the heads of the families called which songs were set to various musical kulapas, or heads of the fighting hordes modes or ragas. called gramanis.  He built a new capital, Nauraspur, in which a large number of musicians were invited to settle. 32. Correct Option: (b)  In his songs, he freely invoked the goddess Explanation: of music and learning, Saraswati. Due to  Statement 3 is incorrect: He stopped sati, his broad approach he came to be called the burning of a widow, unless she herself, ‘Jagat Guru’. He accorded patronage to all, of her own free will, persistently desired it. including Hindu saints and temples. This included grants to Pandharpur, the centre Supplementary notes: of the worship of Vithoba, which became the centre of the Bhakti movement in Social and Educational Reforms by Maharashtra. Akbar  Akbar introduced a number of social and 31. Correct Option: (b) educational reforms. Explanation:  He stopped sati, the burning of a widow, unless she herself, of her own free will,  Statement 1 is incorrect: Tribal chief did persistently desired it. Widows of tender not exercise unlimited powers but had to age who had not shared the bed with their reckon with the tribal organizations. husbands were not to be burnt at all.  Statement 2 is incorrect: Vrajapati was  Widow Remarriage was also legalized. an officer who enjoyed authority over a large stretch of land or pasture ground.  Akbar was against anyone having more than one wife unless the first wife was barren. Supplementary notes:  The age of marriage was raised to 14 Administration in Rig Vedic Period for girls and 16 for boys.  The administrative machinery of  The sale of wines and spirits was the Aryans in the Rig Vedic period functioned with the tribal chief, for restricted. his successful leadership in war, at the  Akbar also revised the educational centre. He was called Rajan. It seems that syllabus, laying more emphasis on in the Rig VedicGS period, the king’s postSCORE had moral education and mathematics, and become hereditary. However, the rajan on secular subjects such as agriculture, was a kind of chief and did not exercise geometry, astronomy, rules of unlimited power, having to reckon government, logic, history, etc. with the tribal organizations.  He also gave patronage to artists, poets,  Several tribal or kin-based assemblies such painters and musicians, so much so that as the sabha, samiti, vidatha, and gana are his court became famous for the galaxy of mentioned in the Rig Veda. They exercised deliberative, military, and religious renowned people known as the navaratna. functions. Even women attended the sabha  However, not all these steps were, however, and vidatha in Rig Vedic times. successful. As we know, the success of social  In the day-to-day administration, the king legislation depends largely on the willing was assisted by a few functionaries. The cooperation of the people. Akbar was living most important of these seems to have been in an age of superstition and it seems that the purohita. his social reforms had only limited success.

15 PTS2020/PPP-21/122019/26

Website ➡ https://upscpdf.com https://t.me/UPSC_PDF Website ➡ https://upscpdf.com https://t.me/UPSC_PDF

33. Correct Option: (c) 34. Correct Option: (a) Explanation: Explanation:  Option (c) is correct:  Statement 2 is incorrect: The Hindus were considered zimmis or protected people, Terminology Feature for which they were forced to pay a tax A. Vishti 4. Forced labour called jizya. B. Kumaramatyas 3. Important officers in Supplementary notes: the Gupta empire Socio-Economic System during the C. Vishayas 2. Part of provincial and Sultanate Period local administration  Delhi Sultans introduced reforms in the system land revenue administration. The lands D. Romaka Sidhanta 1. Book on astronomy were classified into three categories: Supplementary notes:  Iqta land: Lands assigned to officials as iqtas instead of payment for their Life during the Gupta Period services.  There was a general increase in the number of  Khalisa land: Land under the land taxes and various other burdens during direct control of the Sultan and the the Gupta rule for example, Whenever the revenues collected were spent for royal army passed through the countryside, the maintenance of royal court and the local people had to feed it. The peasants royal household. had also to supply animals, food grains,  Inam land: Land assigned or granted furniture, etc., for the maintenance of royal to religious leaders or religious officers on duty in rural areas. institutions.  In central and western India, the  Muslim society remained divided into villagers were also subjected to forced several ethnic and racial groups. The Turks, labor called “Vishti” by the royal army Iranians, Afghans, and Indian Muslims and officials. developed as exclusive groups and there  Gupta bureaucracy. were no intermarriages between them.   The most important officers Hindu converts from lower castes were also in the Gupta empire were the not given equal respect. The Muslim nobles occupied high offices and very rarely the “Kumaramatyas”. Hindu nobles were given a high position in  They were appointed by the king in the the government. home provinces and possibly paid in  The Hindus were considered zimmis or cash. protected people, for which they were  As the Guptas were possibly vaishyas, forced to pay a tax called jizya. In the recruitment was not confined to the beginning, jizya was collected as part of a upper varnas only, but several offices land tax. were combined in the hands of the same  Firoz Tughlaq separated it from the land person, and posts became hereditary. revenue and collected jizya as a separate  This naturally weakened royal control. tax and levied it on Brahmans also.  The empire GSwas divided into divisionsSCORE called bhuktis, and each bhukti was 35. Correct Option: (a) placed under the charge of an uparika. Explanation:  The bhuktis were divided into  districts (vishayas), which were Statement 4 is incorrect: As the people placed under the charge of a had to depend almost entirely on tools and weapons made of stone, they could not found vishayapati. settlements far away from the hilly areas in  The period also witnessed the developments this phase. in the field of mathematics and astronomy. Supplementary notes: For e.g Aryabhatiya was written by Aryabhata. Neolithic or the New Stone Age  “Romaka Sidhanta”,a book on  In the world context, the New Stone or the astronomy was compiled in the same Neolithic age began in 9000 BC but in the period its title indicating that it was Indian subcontinent, it began around the influenced by Greek and Roman ideas. seventh millennium BC. Some important

PTS2020/PPP-21/122019/26 16

Website ➡ https://upscpdf.com https://t.me/UPSC_PDF Website ➡ https://upscpdf.com https://t.me/UPSC_PDF

crops, including wheat and barley, came  The people of the new Stone Age/Neolithic to be cultivated in the subcontinent, and suffered from one great limitation. As they villages were established in this part of the had to depend almost entirely on tools world. and weapons made of stone, they could not found settlements far away from  Perhaps the most fundamental advance that the hilly areas. They could settle only on has affected the course of human life was the the slopes of the hills in rock shelters and domestication of a large number of animals the hilly river valleys. and plants. The domestication of various species of animals produced the specialized pastoralists and the domestication and 36. Correct Option: (d) successful exploitation of various species of wild plants produced a shift towards Explanation: sedentary settlements.  Option (d) is correct:  Some important crops, including wheat Tax Purpose and barley, came to be cultivated in the subcontinent, and villages were A. Sharb 4. Irrigation tax established. B. Kharaj 1. Imposed on 10% of  The earliest Neolithic settlement in the produce Indian subcontinent first developed in C. Zakat 3. Alms for Muslims the west of the Indus. Mehrgarh is one of the largest Neolithic settlements between D. Khams 2. 20% spoils of war the Indus and the Mediterranean and is attributed to 7000 BC. Supplementary notes:  The people of the Neolithic age used tools Economy under Firoz Shah Tughlaq and implements of polished stone. Based on  Firoz Shah Tughlaq is known to have made the types of axes used by Neolithic settlers, the Iqtadari system hereditary which was we notice three important areas of Neolithic introduced in India by Iltutmish. In this settlements—north-western, north-eastern, system, the officers were paid in land grants and southern. The north-western group which were known as Iqtas which were given of Neolithic tools is distinguished by to them in specific territorial boundaries. rectangular axes with a curved cutting edge;  the north-eastern group by polished stone He was the first Sultan to impose Sharb axes with a rectangular butt and occasional (irrigation tax). But at the same time, he shouldered hoes; and the southern group by built the number of canals and also set up a axes with oval sides and pointed butt. separate department of public works.   In the north-west, Kashmiri Neolithic He imposed other taxes like Kharaj on 10% culture was distinguished by its of produce, Zakat as alms for Muslims and dwelling pits, wide range of ceramics, Khams which included 20% of the spoils of the variety of stone and bone tools, and war for the royal treasury. the complete absence of microliths.  Another area from which Neolithic tools 37. Correct Option: (b) have been recovered is situated in the hills of Assam. Neolithic tools have also been Explanation: found in the Garo hills in Meghalaya on the  Statement 1 is incorrect: He was the north-eastern frontierGS of India. SCOREmost famous Shaka ruler.  An important group of Neolithic people Supplementary notes: lived in south India, south of the Godavari River. They usually settled on the tops of Rudradaman I granite hills or on plateaus near the river  The most famous Shaka ruler in India banks. They used stone axes and also a kind was Rudradaman I (AD 130–50). of stone blades.  He ruled not only over Sindh, but also over  During the Neolithic phase, several a substantial part of Gujarat, Konkan, the settlements became acquainted with the Narmada valley, Malwa, and Kathiawar. cultivation of cereals and the domestication of animals. So they needed pots in which  He is famous in history because of the they could store their food grains, and repairs he undertook to improve the also pots for cooking, eating, and drinking. Sudarshana lake in the semi-arid zone Pottery, therefore, first appears in this of Kathiawar which had been in use for phase, with handmade pottery in the irrigation for a long time and dated back to early stage. the Mauryas.

17 PTS2020/PPP-21/122019/26

Website ➡ https://upscpdf.com https://t.me/UPSC_PDF Website ➡ https://upscpdf.com https://t.me/UPSC_PDF

 Rudradaman was a great lover of through the recitation of prayers and Sanskrit. Although he had Central Asian performance of sacrifices but these ancestors, he issued the first-ever long were not accompanied by any ritual or inscription in chaste Sanskrit. sacrificial formulae.  All the earlier longer inscriptions that we Supplementary notes: have in India were composed in Prakrit which had been made the state language by Rig Vedic Gods Ashoka.  The Aryans found it difficult to explain the coming of the rains, the appearance of 38. Correct Option: (b) the sun and the moon, and the existence of the rivers, mountains, and the like. They, Explanation: therefore, personified these natural forces and looked upon them as living beings to  Statement 3 is incorrect: Shivaji did not which they attributed human or animal abolish the zamindari (deshmttkhl) system, attributes. Thus there was no idol but awarded jagirs (mokasa) to his officials. worship at this time. However, Shivaji strictly supervised the mirasdars, that is, those with hereditary  We have a large number of such divinities rights in land. in the Rig Veda:

Supplementary notes: Indra Rain god Shivaji’s Organisation of the Army Agni Fire god  Shivaji preferred to give cash salaries to Varuna Personified as water & supposed to keep the regular soldiers, though sometimes natural order in line the chiefs received revenue grants Soma God of plants, & an intoxicating drink (saranjam). somras named after it  The plunder taken by each soldier during Marut Personified as storm campaigns was strictly accounted for. Aditi Goddess of eternity  The regular army (paga) consisting of Usha Appearance of dawn about 30,000 to 40,000 cavalry, as distinct from the loose auxiliaries (silahdars), were  The women divinities like Aditi and Usha supervised by havaldarswho received fixed were not that prominent during Rig Vedic salaries. times. Given the patriarchal nature of society, the male gods were far more  The forts were carefully supervised, Mavali important than female gods. foot soldiers and gunners being appointed there.  The dominant mode of worshipping the gods was through the recitation of Revenue System prayers and performance of sacrifices.  The revenue system seems to have  Offerings of vegetables, barley, etc., were been patterned on the system of Malik made to gods, but in Rig Vedic times this Ambar. A new revenue assessment was was not accompanied by any ritual or completed by Annaji Datto in 1679. sacrificial formulae.  It is not correct to think that Shivaji abolished the Zamindari (deshmttkhl) 40. Correct Option: (a) system, or that he did not award jagirs (mokasa)GS to his officials. SCORE Explanation: However, Shivaji strictly supervised  Option (a) is correct: the mirasdars, that is, those with hereditary rights in land. List I List II  Shivaji supplemented his income by levying A. Kitab-ul-Hind 1. Alberuni a contribution on the neighbouring Mughal territories. This contribution which came to B. Khazain-ul-Futuh 2. Amir Khusrau one founh of the land revenue, began to be C. Tarikh-i-Firoz Shahi 3. Barani called chauthai (one fourth) or chauth. D. Tabaqat-i-Nasari 4. Minhaj-us- Siraj 39. Correct Option: (b) Supplementary notes: Explanation: Literature in the Sultanate Period  Statement 2 is incorrect: The dominant  The Delhi Sultans patronised learning and mode of worshipping the gods was literature. Many of them had great love for

PTS2020/PPP-21/122019/26 18

Website ➡ https://upscpdf.com https://t.me/UPSC_PDF Website ➡ https://upscpdf.com https://t.me/UPSC_PDF

Arabic and Persian literature Barani’s  There was also a growing tendency in Tarikh-i-Firoz Shahi contains the the hierarchization of offices, indicated history of Tughlaq dynasty. by the use of the prefixes maha and  sarva to the official designations. For e.g. Minhaj-us- Siraj wrote Tabaqat-i- dandanayaka,mahadandanayaka and Nasari, a general history of Muslim mahasarvadandanayaka. dynasties up to c.1260 CE.  Land donation to Brahmanas and  Amir Khusrau (c.1252–1325 CE) was the other religious groups became far more most famous Persian writer of this period. prevalent and institutionalised during  Amir Khusrau’s Khazain-ul-Futuh Gupta period. speaks about Alauddin’s conquests. His  The Gupta period saw the surfacing of famous work, the Tughlaq Nama, deals with feudal development in the form of the fiscal the rise of Ghiyasuddin Tughlaq. and administrative concessions to priests  Alberuni’s Kitab-ul-Hind is the most and administrators through various land famous work wherein he comments on grants. Although, the practice was known Indian sciences, Hindu religious beliefs, since Satvahana times, but it became far more institutionalised during thr rule of customs, and social organization. Guptas.It became a regular and palpable institution. 41. Correct Option: (b)  Religious functionaries were granted land, free of tax, for posterity, and they were Explanation: authorized to collect from the peasants all  Statement 1 is incorrect: There was an the taxes that once went directly to the increase in the tendency to appoint high- emperor. They were also empowered to ranking officers on a hereditary basis. punish criminals.  Statement 3 is incorrect: Visyapati was appointed by the Uparika who was in turn 42. Correct Option: (c) appointed by the Emperor directly. Explanation: Supplementary notes:  Both statements are correct Gupta Administration Supplementary notes:  Gupta empire was divided into a number of regional units. It was a multi-layer Features of Inam Grants under reign of administrative system. Akbar  Akbar separated the inam lands from the  Bhukti was the largest administrative zone jagir and khalisa lands, and divided the in the empire. The earliest known use of this empire into six circles for purposes of grant administrative term is seen in the Allahabad of inam lands and their administration. pillar inscription of Samudragupta.  Two features of the inam grants are  Uparika was the highest regional noteworthy. authorties who were appointed directly by the emperor. They were  First, Akbar made it a deliberate part entrusted with both civil and military of his policy to grant inam lands to all responsibilities. persons, irrespective of their religious faith and beliefs.  The territorial-cum-administrative unit below the BhuktiGS was known as vishayaSCORE  Second, Akbar made it a rule that half equivalent to a locality, close to the size of of the inam land should consist of a district. The vishaya was entrusted to cultivable wasteland. Thus, the inam the district administrator (Vishyapati) holders were encouraged to extend who was appointed by the uparikas. cultivation.  The actual burden of administrations and executive functions was borne by a large 43. Correct Option: (c) number of functionaries who were designated Explanation: amatya,mantra, and sachiva. These were terms for high-ranking functionaries and  Both statements are correct ministers. Supplementary notes:  A significant feature in the administrative system is the tendency Gupta Period to appoint high-ranking officers on a  The Gupta period is remarkable for the hereditary basis. production of secular literature, which

19 PTS2020/PPP-21/122019/26

Website ➡ https://upscpdf.com https://t.me/UPSC_PDF Website ➡ https://upscpdf.com https://t.me/UPSC_PDF

consisted of a fair degree of ornate court the Delhi Sultanate, the advent of Babur poetry. and Humayun being in the nature of an interregnum.  Bhasa was an important poet in the early phase of the Gupta period and  Amongst the foremost contributions of Sher wrote thirteen plays. He wrote in Sanskrit, Shah was his re-establishment of law and but his dramas also contain a substantial order across the length and breadth of his amount of Prakrit. He was the author of a empire. He dealt sternly with robbers and drama called “Dradiracharudatta”, which dacoits and with zamindars who refused to was later refashioned as Mrichchhakatika pay land revenue or disobeyed the orders of or the Little Clay Cart by Shudraka. the government.  The Gupta period also saw the development  Sher Shah paid great attention to the of Sanskrit grammar based on the work improvement of communications in of Panini and Patanjali. This period his kingdom. For e.g. he restored the old is particularly memorable for the imperial road called the Grand Trunk Road, compilation of Amarakosha by from the river Indus in the west to Sonargaon Amarasimha, who was a luminary in in Bengal. He also built a road from Agra to the court of Chandragupta II. This Jodhpur and Chittor, evidently linking up lexicon is learnt by heart by students with the road to the Gujarat seaports. He learning Sanskrit in the traditional way. built a third road from Lahore to Multan. For the convenience of travelers, Sher Shah  Gupta period became particularly famous built a sarai at a distance of every two Kos due to the work of Kalidasa who lived in (about eight km) on these roads. the second half of the fourth and the first half of the fifth century. He was the greatest  Sher Shah also introduced other reforms to poet of classical Sanskrit literature and promote the growth of trade and commerce. wrote Abhijnanashakuntalam which is very In his entire empire, customs duty for goods highly regarded in world literature. was paid only at two places. His attempt to fix standard weights and measures and a uniform standard for coins all over the 44. Correct Option: (a) empire were also helpful for trade and Explanation: commerce.   Statement 1 is incorrect: Sher Shah Sher Shah did not make many changes did not make many changes in the in the administrative divisions administrative divisions prevailing since the prevailing since the Sultanate period. Sultanate period. He apparently continued A number of villages comprised a pargana. the central machinery of administration The pargana was under the charge of the which had developed during the Sultanate shiqdar, who looked after law and order and period. general administration, and the munsif or amil who looked after the collection of land  Statement 2 is incorrect: Sher Shah did revenue. He apparently continued the not initiate any new liberal policies. Jizyah central machinery of administration continued to be collected from the Hindus. as he did not favor leaving too much authority in the hands of ministers. Supplementary notes:  He paid special attention to the land revenue Sher Shah and the Sur Empire (1540- system. He insisted on measurement of the 1555) sown land. A crop rate (called ray) was  Sher Shah ascendedGS the throne of DelhiSCORE at drawn up, laying down the state’s share of the age of 54. His original name was Farid the different types of crops. and his father was a small jagirdar at  Sher Shah did not initiate any new Jaunpur. The title of Sher Khan was given liberal policies. Jizyah continued to be to him by his patron for killing a tiger or for collected from the Hindus. services rendered.  His nobility was drawn almost exclusively  As a ruler, Sher Shah ruled the mightiest from the Afghans. Thus, the state under the empire which had come into existence in Surs remained an Afghan institution based north India since the time of Muhammad oil race and tribe. bin Tughlaq. His empire extended from Bengal to the Indus, excluding Kashmir. In the west, he conquered Malwa, and almost 45. Correct Option: (c) the entire Rajasthan. Explanation:  The Sur Empire may he considered in many  ways as a continuation and culmination of Both statements are correct

PTS2020/PPP-21/122019/26 20

Website ➡ https://upscpdf.com https://t.me/UPSC_PDF Website ➡ https://upscpdf.com https://t.me/UPSC_PDF

Supplementary notes: a permanent one as the state retained the right of modification. Harappan Civilization  Statement 2 is incorrect: Under this  Religious Practices system, the state’s demand was stated in  In Harappa numerous terracotta cash. figurines of women have been found. In Supplementary notes: one figurine, a plant is shown growing out of the embryo of a woman. The Dahsala or Zabti System image probably represents the goddess  of earth, and was intimately connected In 1580, Akbar instituted a new system with the origin and growth of plants. called the dahsala. Under this system, the average produce of different crops The Harappans, therefore, looked upon as well as the average prices prevailing the earth as a fertility goddess. over the last ten (dah) years were  There is also an evidence of worship of calculated. One third of the average male deity. A male deity, “the prototype produce was the state share. The state of the historic Siva,” is portrayed on a demand was, however, stated in cash. seal with three faces, seated on a low This was done by converting the state share throne in the typical posture of a Yogi, into money on the basis of a schedule of with two animals on each side - elephant average prices over the past ten years. and tiger on right and rhinoceros and  Later, a further improvement was made. buffalo on left, and two deer standing Not only were local prices taken into under the throne. The depiction shows account, parganas having the same type Siva as Pashupati. of productivity were grouped into separate  There are the evidnces of the assessment circles. Thus, the peasant prevalence of the phallus worship, was required to pay on the basis of local which in later times became so productivity as well as local prices. intimately connected with Shiva.  There were a number of advantages of Numerous symbols of the phallus and this system. As soon as the area sown by female sex organs made of stone have the peasant had been measured by means been found in Harappa, and were of the bamboos linked with iron rings, the possibly meant for worship. peasants as well as the state knew what the  The people of the Indus region also dues were. The peasant was given remission worshipped trees. The depiction of a in the land revenue if crops failed on deity is represented on a seal amidst account of drought, floods, etc. The system branches of the pipal. of measurement and the assessment based upon it is called the zabti system. Akbar  Animals like one horned unicorn, introduced this system in the area from humped bull etc. were also Lahore to Allahabad and in Malwa and worshipped in Harappan times. Gujarat. The dahsala system was a further  Script development of the zabti system.  The Harappan script is not  The dahsala was not a ten-year alphabetical but largely settlement. Nor was it a permanent pictographic and has not been one with the state retaining the right deciphered yet. to modify it. However, with some changes, Akbar’s settlement remained the basis of the  Some scholars opine that Harappan land revenue system of the Mughal Empire inscriptions present 3 logosyllablic till the end or the seventeenth century. writing system,GS where a sequence SCORE of  two or more signs would represent a The zabti system is associated with Raja complete word, a syllable or a sound and Todar Mal, and is sometimes called Todar sometimes even a sentence of several Mal’s bandobast. Todar Mal was a brilliant words and grammatical indicators. revenue officer who had first served under Sher Shah.  The script was written from right to left.  When the inscription was of more than The peasants were allowed to choose between one line it could be first line from right to zabti and batai; under certain conditions. left and second from left to right. Thus, such a choice was given when the crops had been ruined. Under batai, the peasants were given the choice of 46. Correct Option: (d) paying in cash or in kind, though the state preferred cash. In case of crops such Explanation: as cotton, indigo, oil seeds, sugarcane, etc.,  Statement 1 is incorrect: Dahsala was the state demand was invariably in cash. neither a system of ten year settlement nor Hence, these were called cash crops.

21 PTS2020/PPP-21/122019/26

Website ➡ https://upscpdf.com https://t.me/UPSC_PDF Website ➡ https://upscpdf.com https://t.me/UPSC_PDF

47. Correct Option: (b)  Khizr Khan captured Delhi by defeating Daulat Khan of the Tughlaq Dynasty and Explanation: founded the Sayyid dynasty in c.1414 CE.  Option (b) is correct However, he did not assume the title of the Sultan but was comfortable with Rayat- Supplementary notes: i-Ala. The Kushans  Khizr Khan was succeeded by his son, Mubarak Shah.  The Kushans are also called Yuechis  or Tocharians. The Tocharians were Muhammad Shah was the third monarch of considered to be the same as the Scythians. the Sayyid dynasty which ruled the Delhi The Kushans were one of the five clans into Sultanate. He succeeded his uncle, Mubarak which the Yuechi tribe was divided. Shah to the throne.   A nomadic people from the steppes Muhammad Shah died in 1445 and was succeeded by his son Alam Shah. of north Central Asia living in the neighbourhood of China, the Kushans  He proved to be an incompetent Sultan and first occupied Bactria or north the weakest of the Sayyid princes. Afghanistan where they displaced the  Alam Shah’s Wazir, Hamid Khan, invited Shakas. Gradually they moved to the Bahlol Lodhi to take charge of the army Kabul valley and seized Gandhara by and after realizing that it would be difficult crossing the Hindu Kush, replacing the to continue as Sultan, Alam Shah left for rule of the Greeks and Parthians in these Badaun. areas. They eventually established their authority over the lower Indus basin and the greater part of the Gangetic basin. 49. Correct Option: (d)  Their empire extended from the Oxus to Explanation: the Ganges, from Khorasan in Central  Option (d) is correct Asia to Pataliputra in Bihar.  A substantial part of Central Asia Supplementary notes: now included in the Commonwealth of Inscriptions Independent States (in the former USSR),  a portion of Iran, a portion of Afghanistan, One of the most important reliable sources of history writing is inscription. An inscription, almost the whole of Pakistan, and almost being a contemporary document, is free the whole of northern India were brought from later interpolations. It comes in the under one rule by the Kushans. form it was composed in and engraved for  Because of this, the in India the first time. It is almost impossible to add is sometimes called a Central Asian empire. something to it at a later stage. In any case, the empire created a unique  The study of inscriptions is called opportunity for the interaction of peoples epigraphy. and cultures, and the process gave rise to a new type of culture which embraced nine  Ashokan inscriptions are considered to modern countries. be the earliest. These are found written in four scripts.  48. Correct Option: (a) In his empire in Afghanistan he used GS SCOREAramaic and Greek scripts for his Explanation: edicts.  Option (a) is correct: The correct  In the Pakistan region Kharoshthi chronology is: Khizr Khan - Mubarak Shah script was used. - Muhammad Shah - Alauddin Alam Shah  The Brahmi script was used for the rest Supplementary notes: of his empire from Kalsi in the north in Uttaranchal upto Mysore in the south. Sayyid Dynasty  The inscriptions of Ashoka are a class in  Timur, the Mongol leader of Central Asia, itself. These were recorded in different years head of Chagatai Turks invaded India in of his reign and are called edicts because c.1398 CE. they are in the form of the king’s order or desire. They also give a glimpse of Ashoka’s  Before his departure from India, Timur image and personality as a benevolent king appointed Khizr Khan as the governor of concerned with the welfare of not only his Multan. subjects but also of the whole humanity.

PTS2020/PPP-21/122019/26 22

Website ➡ https://upscpdf.com https://t.me/UPSC_PDF Website ➡ https://upscpdf.com https://t.me/UPSC_PDF

50. Correct Option: (b) the Harappans who were urbanized on the basis of the produce from the flood plains in Explanation: the Indus Valley.  Statement 1 is incorrect: Amalguzar was  The Chalcolithic cultures are all officer-in-charge for the assessment and characterized by painted ceramic, usually collection of land revenue in pargana. black-on-red, a specialized blade and Supplementary notes: flake industry of the siliceous material like chalcedony and chert, and copper and Organization of Local Government under bronze tools, though on a restricted scale. reign of Akbar Their economy was based on subsistence agriculture, stock-raising and hunting and  Akbar made hardly any changes in the fishing. organization of local government.  In India, settlements relating to the  The pargana (tehsil) and the sarkar (district) continued as before. The chief Chalcolithic phase are found in southeastern officers of the sarkar were the faujdar Rajasthan, the western part of MP, western and the amalguzar, the former being in Maharashtra, and in southern and eastern charge of law and order, and the latter India. responsible for the assessment and  The people of the Chalcolithic phase collection of the land revenue. use different types of pottery, one  The territories of the empire were divided of which is called black-and-red and into jagir, khalisa and inam. seems to have been widely prevalent from nearly 2000 BC onwards and OCP  Income from khalisa villages went directly (Ochre-Colored Pottery) which is a red- to the royal exchequer. slipped ware often painted in black  The inam lands were those which were and largely in vase forms. allotted to learned and religious men.  In Maharashtra, people buried their dead  Jagirs were allotted to nobles and members in urns beneath the floor of their house in of the royal family including the queens. the north-to-south position. They did not use separate cemeteries for this purpose, as  The amalguzar was required to exercise a was the case with the Harappans. Pots and general supervision over all types of holdings some copper objects were deposited in the so that the imperial rules and regulations graves obviously for the use of the dead in for the assessment and collection of land the next world. revenue were followed uniformly.  Terracotta figures of women suggest that the Chalcolithic people venerated the 51. Correct Option: (a) mother goddess, and some unbaked nude Explanation: clay figurines were also used for worship.  A kind of settlement hierarchy is visible in  Statement 1 is incorrect: The Chalcolithic people mostly used copper objects along several Jorwe settlements of Maharashtra. with stone objects and occasionally used of Some of them are as large as twenty low-grade bronze. hectares, but others encompass only five hectares and even less. This would imply Supplementary notes: two-tier habitations. Chalcolithic Age  Both the settlement pattern and burial practices suggest the beginnings of  The end of theGS Neolithic period sawSCORE the social inequalities in Chalcolithic use of metals. The metal first used was society. copper, and several cultures were based on the use of copper and stone  The Chalcolithic people domesticated implements. Such a culture is called cattle, sheep/goats. In all probability, the Chalcolithic, which means the copper domesticated animals were slaughtered – stone phase. for food and not used for milk and dairy products.  Technologically, the Chalcolithic stage is applied to the pre-Harappan phase. However, in various parts of India the 52. Correct Option: (b) Chalcolithic cultures followed the Bronze Age Harappa culture. Explanation:  The Chalcolithic people were primarily  Statement 2 is incorrect: Balban is known rural communities spread over a wide to have been deeply racist and excluded area with hilly land and rivers unlike non-Turks from administration.

23 PTS2020/PPP-21/122019/26

Website ➡ https://upscpdf.com https://t.me/UPSC_PDF Website ➡ https://upscpdf.com https://t.me/UPSC_PDF

 Statement 3 is incorrect: He followed the Trade and Agriculture policy of “Blood and Iron” to gain control in  The coming of the Central Asian people the region. established intimate contacts between Supplementary notes: Central Asia and India. India received a great fund of gold from the Altai Mountains Reign of Balban in Central Asia.  Balban ascended the throne of Delhi  The Kushans controlled the Silk Route, Sultanate in 1265. which started from China and passed through their empire in Central Asia and  He administered justice with extreme Afghanistan to Iran, and western Asia impartiality but was deeply racist which formed part of the Roman empire in and excluded non-Turks from the eastern Mediterranean zone. administration. He stood forth as the  champion of the Turkish nobility. It is significant that the Kushans were the first rulers in India to issue gold  Indian Muslims were not given important coins on a wide scale. The Kushans also posts in the government. He appointed spies promoted agriculture. to monitor the activities of the nobles. Religious Developments  Balban maintained a magnificent court  Some rulers and others from Central Asia and also introduced the Persian festival of adopted Vaishnavism, which means the Nauroz. worship of Vishnu, the god of protection  During his reign, the law and order situation and preservation. The Greek ambassador in the area around Delhi and in the doab called Heliodorus set up a pillar in honour had deteriorated. of Vasudeva at Besnagar near Vidisa (headquarters of Vidisa district) in MP  In the Ganga-Jamuna doab and Awadh, around the middle of the second century the roads were infested with robbers and BC. A few other rulers adopted Buddhism. dacoits, so much so that communication with The Origin of Mahayana Buddhism the eastern areas had become difficult.  Due to the large influx of people from  He followed the policy of “Blood and Central Asia, Buddhism was especially Iron” to ouster the zamindars affected. Discipline became so lax that who had set up forts in the area by some renunciated or even deserted the defying the government and the religious order or the samgha and resume Mewatis who had become so bold as to the householder’s life. This new form of plunder people up to the outskirts of Buddhism came to be called Mahayana or the city. the Great Vehicle.  These were ruthlessly hurled down and killed, the forests around Delhi cut down, 54. Correct Option: (c) and many military outposts (thanas) were established there. Explanation:  Both statements are correct 53. Correct Option: (a) Supplementary notes: Explanation: Chola Government  Statement 4 GSis incorrect: Mahayana SCORE  The king was the most important person in Buddhism has been originated in India due the Chola administration. All authority to Central Asian invasion. rested in his hands, but he had a council of ministers-to advise him. The Supplementary notes: kings often went on tours in order to oversee the administration. Cultural Consequences of Central Asia Invasion  The Cholas maintained a large share of elephants, cavalry and infantry which were Better Cavalry called the three limbs of the army. The  They introduced better cavalry and the use infantry was generally armed with spears. Most of the kings had bodyguards who were of the riding horse on a large scale.They sworn to defend the kings even at the cost popularized the use of reins and saddles, of their lives. which appear in the Buddhist sculpture of the second and third centuries AD. The  The Chola state included area of central Shakas and the Kushans were excellent control, and loosely administered areas horsemen. under different types of local control.

PTS2020/PPP-21/122019/26 24

Website ➡ https://upscpdf.com https://t.me/UPSC_PDF Website ➡ https://upscpdf.com https://t.me/UPSC_PDF

 The state was interpersed with hill 56. Correct Option: (a) people and tribals. The basic unit of administration was the Nadu which Explanation: consisted of a number of villages  Option (a) is incorrect: Alai Darwaza having close kinship ties and other was built by Alauddin Khalji in 1311 and is close associations. made of red sandstone.  The number of nadu increased as fresh lands Supplementary notes: were brought under cultivation by means ofirrigation works such as ponds, wells, etc., Sultanate art and architecture and by converting hill or tribal people into  The art and architecture of the Delhi agriculturists. Sultanate period were distinct from the  Grants to Brahmans and temples increased, Indian style. The Turks introduced arches, both of which helped in expanding domes, lofty towers, or minarets and cultivation. decorations using the Arabic script.  The most magnificent building of the 13th 55. Correct Option: (c) century was started the Qutub Minar, which was founded by Aibak and completed Explanation: by Iltutmish.  Option (c) is correct:  Later, Alauddin Khalji added an entrance to the Qutub Minar called Books Writers Alai Darwaza. The dome of this arch A. Malavikagnimitram 3. Kalidasa was built on scientific lines.  B. Mudrarakshasha 4. Vishakhadatta The buildings of the Tughlaq period were constructed by combining arch and dome. C. Vikramankadevacharita 1. Bilhana The palace complex called Tughlaqabad D. Harshacharita 2. Banabhatta with its beautiful lake was built during the period of Ghyasuddin Tughlaq. Supplementary notes:  Muhammad bin Tughlaq built the tomb of Literary Sources Ghyasuddin on a high platform.  The works of Kalidasa comprise kavyas  The Kotla fort at Delhi was the creation of and dramas, the most famous of which is Firoz Tughlaq. Abhijnanashakuntalam. Besides being great creative compositions, they provide us 57. Correct Option: (b) with glimpses of the social and cultural life of the Guptas. Explanation:  Kalidasa’s Malavikagnimitram is  Statement 2 is incorrect: Gupta period based on some events of the reign of was poor in terms of architecture. Pusyamitra Sunga, a dynasty which followed the Mauryas. Supplementary notes:  Mudrarakshasha, a play written by Gupta Architecture Vishakhadatta, also gives a glimpse of  The Gupta period was poor in terms of society and culture.GS SCOREarchitecture.  Banabhatta’s Harshacharita throws  All that we find are a few temples made light on many historical facts about of brick in UP and a stone temple. which we could not have known  The brick temples of Bhitargaon in otherwise. Kanpur, Bhitari in Ghazipur, and  Bilhana’s Vikramankadevacharita Deogarh in Jhansi may be mentioned. describes the victories of the later  The Buddhist University at Nalanda Chalukya king Vikramaditya. was set up in the fifth century, and its  Rajatarangini by Kalhana is the best earliest structure, made of brick, relates illustration of history writing appreciated to this period. by modern historians. His critical method of Gupta Art historical research and impartial treatment of the historical facts have earned him a  The Gupta period is called the Golden Age great respect among the modern historians. of ancient India.

25 PTS2020/PPP-21/122019/26

Website ➡ https://upscpdf.com https://t.me/UPSC_PDF Website ➡ https://upscpdf.com https://t.me/UPSC_PDF

 Guptas possessed a large quantity of gold Supplementary notes: and they issued the largest number of gold coins which were called “dinaras” in their Cultural Contribution of Deccan States inscriptions.  The Deccan states had a number of cultural  After the conquest of Gujarat, the Guptas contributions to their credit. Ali Adil Shah issued a large number of silver coins mainly (d. 1580) loved to hold discussions with for local exchange, in which silver occupied Hindu and Muslim saints and was called an important position under the Western a Sufi. He invited Catholic missionaries to Kshatrapas. his court, even before Akbar had done so. He had an excellent Library to which he  In contrast to those of the Kushans, appointed the well-known Sanskrit scholar, the Gupta copper coins are very few in Waman Pandit. number. This would suggest that the use of money did not touch the common  The successor of Ali Adil Shah, Ibrahim Adil people as much as it did under the Shah II (1580-1627) was deeply interested Kushans in music, and composed a book called Kitab- i-Nauras in which songs were set to various  Both Samudragupta and Chandragupta II musical modes or ragas. He built a new were patrons of art and literature. capital, Nauraspur, in which a large number  Samudragupta is represented on of musicians were invited to settle. his coins playing the lute (vina),  and Chandragupta II is credited Sultan Muhammad Quli Qutub Shah, a with maintaining in his court nine contemporary of Akbar, was very fond luminaries. of literature and architecture. He wrote in Dakhani Urdu, Persian and Telugu and  Buddhism gave great impetus to art in has left an extensive diwan or collection. He Maurya and post-Maurya times. was the first to introduce a secular note in  During the Gupta period a life-size poetry. Apart from the praise of God and the copper image of the Buddha of more Prophet, he wrote about nature, love, and than 6 feet was made. It was discovered the social life of his day. at Sultanganj near Bhagalpur, and is  In the field of architecture, Muhammad now displayed in Birmingham. Quli Qutub Shah constructed many  During the Gupta period beautiful buildings, the most famous of which images of the Buddha were is the Char Minar. Completed in 1591- fashioned at and Mathura, 92, it stood at the centre of the new city of but the finest specimens of Buddhist Hyderabad founded by Muhammad Quli art in Gupta times are the Ajanta Qutub Shah. It has four lofty arches, facing paintings. the four directions. Its chief beauty is the four minarets which are four storied and are  They depict various events in the life 48 metre high. The double screen of arches of Gautama Buddha and the previous has fine carvings. Buddhas whose birth stories are related in the Jatakas.  The rulers of Bijapur consistently maintained  As the Guptas supported Brahmanism, a high standard and an impeccable taste images of Vishnu, Shiva, and some other in architecture. The most famous Bijapur Hindu gods were fashioned for the first time buildings of the period are the Ibrahim during their period. Rauza and the Gal Gumbaz.   At many places,GS the entire pantheon SCORE The former was a mausoleum for Ibrahim is portrayed with the chief god at the Adil Shah and shows the style at its best. centre surrounded by his retainers and The Gol Gumbaz which was built in subordinates. 1660 has the largest single dome ever constructed. All its proportions are  The leading god is represented as large in harmonious, the large dome being balanced size, with his retainers and subordinate by tall, tapering minarets at the corner. It is gods drawn on a smaller scale. said that a whisper at one side of the huge  This reflects clear social hierarchy and main room can be heard clearly at the other discrimination. end.

58. Correct Option: (d) 59. Correct Option: (d) Explanation: Explanation:  All statements are correct  All statements are correct

PTS2020/PPP-21/122019/26 26

Website ➡ https://upscpdf.com https://t.me/UPSC_PDF Website ➡ https://upscpdf.com https://t.me/UPSC_PDF

Supplementary notes:  Jalaluddin followed the policy of tolerance to win the goodwill of the The Kalabhra Revolt nobility by retaining the earlier  It is one of the important revolt of the sixth nobility in his administration. century. The Kalabhras seem to have been  For instance, Malik Chhajju, nephew of a tribal people who captured power, Balban, was allowed to remain the governor particularly at the cost of the Cholas, of Kara. When Chhajju revolted, it was and ruled for seventy five years. suppressed, but he was pardoned.  Their rule also affected the Pallavas as well  In the year 1292 CE, when Malik Chhajju as their neighbouring contemporaries. revolted for the second time, he was replaced  The Kalabhras are called evil rulers, by his nephew and son-in-law, Alauddin who overthrew innumerable kings and Khalji. established their hold on the Tamil land. The Kalabhra revolt was a powerful 61. Correct Option: (c) peasant protest directed against the landed brahmanas. Explanation:  They put an end to the brahmadeya rights  Option (c) is incorrect granted to the brahmanas in numerous villages. Supplementary notes:  It appears that the Kalabhras were of Social Developments in the Gupta Buddhist persuasion as they patronized Period Buddhist monasteries.  Guptas, who were originally Vaishyas,  The Kalabhras’ revolt was so widespread became the great supporters of the that it was be quelled only through Brahmanical order who in turn legitimized the joint efforts of the Pandyas, their position by looking upon Guptas as the Pallavas, and the Chalukyas of Kshatriyas. Badami.  Shudras  By the last quarter of the sixth century,  Castes were proliferated into numerous according to a tradition, the Kalabhras had sub-castes during this period and the imprisoned the Chola, the Pandya, and position of Shudras improved during the Chera kings, which underlines how this period. formidable their revolt was.  They were now permitted to listen  The confederacy of the kings formed against to recitations of the , the the Kalabhras, who had revoked the land , and the Puranas. grants made to the brahmanas, shows that the revolt was directed against the  They could also worship a new god existing social and political order in called Krishna and were also permitted south India. to perform certain domestic rites which naturally meant fees for the priests. This can all be linked to some 60. Correct Option: (c) improvement in the economic status of the Shudras. Explanation:  Untouchables  Both statements are correct  During this period, the number of Supplementary notes:GS SCOREuntouchables increased, especially the Jalaluddin Khalji chandalas.  Fa-hsien mentions chandalas living  Jalaluddin Khalji laid the foundation of the outside the village and dealing in meat Khalji dynasty in India in the year 1290. and flesh.  During Balban’s reign, Jalaluddin had been  Women the warden of the marches in north-west and had fought many successful battles  Like the shudras, women were also against the Mongols. allowed to listen to the Ramayana, the Mahabharata, and the Puranas, and  He was generous and also the first were advised to worship Krishna. sultan of the Delhi Sultanate to have a benevolent attitude towards Hindus.  Women of Upper Varna  He clearly put forward the view that the  The Gupta period is marked with state should be based on the willing support the subordination of women of of the governed. upper varnas.

27 PTS2020/PPP-21/122019/26

Website ➡ https://upscpdf.com https://t.me/UPSC_PDF Website ➡ https://upscpdf.com https://t.me/UPSC_PDF

 The principal reason for the  By the 17th century, several of these nayakas subordination of women of the upper such as those of Tanjore and Madurai varnas was their complete dependence became independent and established on men for their livelihood, and lack separate states. of proprietary rights.  The position of women was not much  The only property they could hold on improved. was the “stridhana” which included  Large number of women were employed presents received by the bride not in the royal palaces as dancers, domestic only from her parents’ side but also servants, and palanquin bearers. from her parents-in-law at marriage.  Sahagaman, i.e., Sati was honoured.  A daughter was not allowed to inherit landed property in  The practice of devadasi (attachment the patriarchal communities of of dancing girls to temples) was India. prevelant.   Women of Lower Varna Polygamy was prevalent among the royal families.  Women of the two lower varnas were free to earn their livelihood, which gave them considerable freedom, 63. Correct Option: (c) but this was denied to women of the upper varnas. Explanation:   In a patriarchal setup, higher order Option (c) is correct: began to treat women as items of List-I List-II property, to such a degree that a woman was expected to follow her husband to A. Mahamatras 4. District Officials the next world. B. Gaulmika 3. Head of Military  The first example of the immolation of Regiment a widow after the death of her husband C. Maharashtrikas 1. High Officials of occurred during the Gupta period in AD administrative 510. division  Brahmana’s material conditions D. Senapati 2. Provincial Governor improved immensely as the recipient of agraharas. They were now associated Supplementary notes: with agrarian life as an owner of the Pattern of Administration during land or a landed intermediary, though Satavahana Phase not as a direct tiller of the soil.  The Satavahanas retained some administrative structures of Ashokan 62. Correct Option: (a) times. Explanation:  Their district was called ahara, as it was known in the time of Ashoka, and their  Statement 2 is incorrect: The practices officials were known as amatyas and such as devadasi and sati were honoured mahamatras, as was the case in Maurya during this period. times. Supplementary notes:  However, their administrative divisions were also called rashtra, and their high Social life duringGS the Vijayanagar SCORE officials were styled maharashtrikas. Period  We notice certain military and feudal traits  One of the important characteristics of the in the administration of the Satavahanas. Vijayanagar administration was the ‘Amara It is significant that the senapati was – nayaka’ system. The top-grade officers appointed provincial governor. of the army were known as Nayaks or  Palaiyagars or Poligars. As the tribal people in the Deccan were not thoroughly brahmanized and reconciled  Nayakas officers were granted land to the new rule, it was necessary to keep (called amaram) in lieu of their services them under strong military control. The while soldiers were usually paid in administration in the rural areas was cash. placed in the hands of a gaulmika, the  head of a military regiment consisting The nayaka was responsible for expanding of nine chariots, nine elephants, twenty- agricultural activities in his amaram (area). five horses, and forty-five foot-soldiers. The The nayaka was also the commander of the head of this regiment was posted in the forts. countryside to maintain peace and order.

PTS2020/PPP-21/122019/26 28

Website ➡ https://upscpdf.com https://t.me/UPSC_PDF Website ➡ https://upscpdf.com https://t.me/UPSC_PDF

64. Correct Option: (b) ranges, which are a southward continuation of the Himalayas, could Explanation: be crossed through the Khyber,  Option (b) is correct: Bolan, and Gomal passes. The Sulaiman ranges are joined southward List I List II in Baluchistan by the Kirthar ranges which could be crossed through the Bolan A. Polaj 3. Land always under Pass. Through these passes, two-way cultivation traffic between India and Central Asia has continued from prehistoric B. Parati 2. Land which is times onwards. temporarily uncultivated  The foothills of the Himalayas lent C. Chachar 4. Land left fallow for 2-3 themselves to easier clearance than years the jungles on the alluvial soil of the plains. It was easy to cross rivers in D. Banjar 1. Land uncultivated for 5 these areas because of their narrower years or more width, and hence the earliest routes Supplementary notes: skirted along the foothills of the Himalayas from the west to the east. Ascertainment of Produce per Bigha It was therefore natural that the  In fixing the land revenue, continuity of earliest agricultural settlements cultivation was taken into account. were founded in the foothills and uplands, and trade routes followed the  Land which remained under cultivation terai route. almost every year was called polaj.  Proceeding from west to east we  When it remained uncultivated it was find the annual rainfall gradually called parati (fallow). Parati land paid increasing from 25 cm to over 250 at the full (polaj) rate when it was cm. The Indus vegetation based on 25 to cultivated. 37 cm rainfall and possibly the western  Land which had been fallow for two to three Gangetic vegetation based on 37 to 60 years was called chachar, and banjar if cm rainfall could be cleared with stone longer than that. and copper implements and made fit for cultivation, but this was not possible in  These were assessed at concessional rates, the case of the mid-Gangetic vegetation the revenue demand gradually rising till the based on 60 to 125 cm rainfall, and full or polaj rate was paid in the fifth or the certainly not in the case of the lower eighth year. In this way, the state helped in Gangetic and Brahmaputra vegetation bringing virgin and uncultivated wasteland based on 125 to 250 cm rainfall. The under cultivation. thickly forested areas, which also  Land was classified further into good, had hard soil, could be cleared only middling and bad. One third of the average with the aid of iron implements produce was the state demand, but it varied which became available at a much according to the productivity of the land, later stage. Therefore, the natural the method of assessment, etc. resources of the less rainy western area were utilized first. 65. Correct Option: (d) 66. Correct Option: (b) Explanation: GS SCORE Explanation:  All statements are correct  Option (b) is correct: Supplementary notes: List I List II Geographical Setting A. Ghiyasuddin 3. First sultan to  Large-scale human settlements generally Tughlaq start irrigation spread from west to east. works  Reasons for such migration pattern are: B. Muhmmad bin 4. Diwan-i-amirkohi  The Himalayas are sufficiently high to Tughluq shield India against invasions from the north. This was especially true in pre- C. Ibn Batuta 1. Safarnama Rehla industrial times when communications D. Khan-i-Jahan 2. A Telugu Brahmin were very difficult. However, on the north-west, the Sulaiman mountain Maqbal convert Wazir

29 PTS2020/PPP-21/122019/26

Website ➡ https://upscpdf.com https://t.me/UPSC_PDF Website ➡ https://upscpdf.com https://t.me/UPSC_PDF

Supplementary notes:  There is no reference to the teachings of the Buddha in the Greek and Aramaic Tughlaq Reign versions.  Ghiyasuddin Tughlaq was the first  Ashoka’s inscriptions do not contain sultan to start irrigation works. certain key ideas associated with the  Muhammad bin Tughluq set up a new Buddha’s teaching, such as the Eight- department of Agriculture, “Diwan- Fold Path, the Four Noble Truths, and i-amirkohi”. He launched a scheme by the goal of Nibbana. which takkavi loans (loans for cultivation)  were given to the farmers to buy seed and to However, there is definite Buddhist extend cultivation. core ideology in his dhamma-focussed inscriptions.  “Safarnama Rehla” was authored by Ibn Batuta which is one of the main sources  He was the first Indian king to speak directly of the information regarding the reign of to the people through his inscriptions which Muhammad bin Tughluq in India. carry royal orders. The inscriptions throw light on Ashoka’s career, his external  Firoz Shah Tughlaq had the unique and domestic polices, and the extent of distinction of being chosen as the Sultan by his empire. the nobles. So, his policy aimed to appease the nobles, the army, and theologians.  He appointed Khan-i-Jahan Maqbal, 68. Correct Option: (d) a Telugu Brahmin convert, as Wazir Explanation: (prime minister). The Wazir helped the Sultan in his administration and maintained  Statement 1 is incorrect: Raziya, with the the prestige of the Sultanate during this support of amirs of Delhi, seized the throne period. of the Delhi Sultanate from Firuz Shah, the eldest son of Iltutmish who was supported 67. Correct Option: (c) by the nobles.  Statement 2 is incorrect: Altuniya Explanation: (governor of Lahore) is known to have  Both statements are correct imprisoned Razia at Qila Mubarak in Bathinda. Supplementary notes: Supplementary notes: Ashoka’s edicts  There are a total of 33 inscriptions primarily Death of Iltutmish and his succession classified into Major Rock Edicts, Minor  Iltutmish did not consider any of his sons Rock Edicts, Separate Rock Edicts, Major worthy of the throne, he declared his Pillar Edicts, and Minor Pillar Edicts. daughter Razia as successor, deviating from  The edicts of the earlier half of his reign normal practice. were inscribed on conveniently located  However, the eldest son of Iltutmish was rock surfaces and were widely distributed put on the throne by nobles who wanted a throughout the empire, especially in the puppet ruler. areas of public settlements where people can easily read these edicts. Hence these  Raziya, with the support of amirs of are referred to as the Minor and Major Rock Delhi, seized the throne of the Delhi Edicts. GS SCORESultanate and Firuz Shah was put to death.  Pillar edicts were mostly inscribed in the latter parts of his reign and were on well-  She was the first and only female Muslim polished sandstone monolithic pillars. ruler of medieval India.  These sandstones were quarried from the  Her attempts to create a counter nobility of sites of Chunar, near Varanasi. non-Turks invited the wrath of the Turkish  Each pillar was surmounted with a finely amirs. sculpted animal capital and was largely  They were particularly incensed over her confined to the Ganges plain. decision to appoint an Abyssinian slave,  In his inscriptions, Brahmi, Kharosthi, Malik Jamaluddin Yaqut, to important Prakrit, Aramaic, and Greek languages office of the Amir-i-akhur (Superintendent have been used. of royal horses).  These inscriptions were deciphered by  The recruitment of a few other non-Turks to James Prinsep in 1837 CE. important posts further inflamed matters.

PTS2020/PPP-21/122019/26 30

Website ➡ https://upscpdf.com https://t.me/UPSC_PDF Website ➡ https://upscpdf.com https://t.me/UPSC_PDF

 Among others, Altuniya, the governor of  Most seals have inscriptions in a Lahore was first to create trouble but was pictographic script that is yet to be defeated. deciphered.  In the year 1240 CE, a serious rebellion  Seals were primarily used for commercial broke out in Sirhind (Bhatinda) under purposes and helped in communication. Altuniya.  Some seals with a hole on them have  Raziya, alongside Yaqut, marched been found on dead bodies. This indicates against Altuniya, but on the way, they might have used as amulets. Turkish followers of Altuniya murdered  Yaqut and imprisoned Raziya at Qila Mathematical images have also been Mubarak in Bathinda. found on some seals, which might have been used for educational purposes as  In the meantime, the Turkish nobles put well. Bahram, another son of Iltutmish, on the throne.  Bronze Figures:  However, Raziya won over her captor,  The Harappan civilization saw a wide Altuniya, and after marrying him proceeded scale practice of bronze casting. The to Delhi. But she was defeated and killed on bronze statues were made using “lost the way by Bahram Shah. wax technique” or “Cire Perdue”. Bronze dancing girl of Mohenjodaro, bronze bull of Kalibangan, etc. are examples of 69. Correct Option: (c) bronze figures. Explanation:  Terracotta:  Statement 1 is incorrect: While most  Terracotta refers to the use of fire baked of the Harappan seals were square, that clay for making sculptures. triangular, rectangular and circular seals  have also been found. Also besides steatite, Compared to the bronze figures, chert, copper and terracotta seals have also the terracotta sculptures are less been found. in number and crude in shape and form.  Statement 2 is incorrect: Compared to the bronze figures, the terracotta sculptures  Pottery: are less in number.  The potteries found at the excavation  Statement 3 is incorrect: Plain Pottery sites can be broadly classified into and Red-and-black pottery is associated two kinds – plain pottery and painted with Indus Valley Civilization. pottery. The painted pottery is also known as Red and Black pottery. Supplementary notes:  Most of the potteries that have been Sculptures of Harappan Civilization found are very fine wheel-made wares,  The Harappan sculptors were extremely with a very few being handmade. adept at handling three dimensional  Ornaments: volumes. The most commonly found were seals, bronze figures and potteries.  The Harappans used a large variety of materials, from precious metals and  Seals: GS SCOREgemstones to bones and even baked  Archaeologists have found numerous clay, to make ornaments. seals of different shapes and sizes all  Both men and women wore ornaments across the excavation sites. like necklaces, fillets, armlets and finger  While most seals are square, it was rings. Girdles, earrings and anklets were found that triangular, rectangular worn only by women. and circular seals were also used.  The people of the time were conscious of Steatite, a soft stone found in the fashion as well, as can be inferred from river beds, was although the most the different styles of hair and beard. common material used to make seals, yet agate, chert, copper, faience and terracotta seals have 70. Correct Option: (d) also been found. Explanation:  Some instances of copper, gold and ivory seals have also been found.  All statements are correct

31 PTS2020/PPP-21/122019/26

Website ➡ https://upscpdf.com https://t.me/UPSC_PDF Website ➡ https://upscpdf.com https://t.me/UPSC_PDF

Supplementary notes: Supplementary notes: Mansabdari System and the Army Megaliths  Akbar would not have been able to  The beginnings of the megalithic culture expand his empire and maintain his can be traced to 1000 BC. hold over it without a strong army.  For this purpose, it was necessary for Megaliths were people who claimed him to organize the nobility as well as the upland portions of the peninsula his army. Akbar realized both these up to the second century BC and their objectives by means of the mansabdari concentration seems to be in eastern system. Andhra and in the Tamil Nadu region of the peninsula.  Under this system, every officer was assigned a rank (mansab). The lowest  They are known not from their actual tank was 10, and the highest was 5000 for settlements which are rare, but from their the nobles. Princes of the blood received funerary structures. higher mansabs.  These were predominantly hunters  The ranks or mansabs were divided into and did not practice an advanced type two-zat and sawar. The word zat means of agriculture which is evident from personal status of a person and also the the fewer number of agricultural tools salary due to him. The sawar rank indicated buried in their graves than those meant the number of cavalry men (sawars) a person for fighting and hunting. was required to maintain.  The graves are called megaliths because  A person who was required to maintain as many sawars as his zat rank was placed they were encircled by large pieces of stone. in the first category of zat rank; if he  By the second century BC, the megalithic maintained half or more, then in the second people had moved from the uplands into category and if he maintained less than half fertile river basins and reclaimed marshy then in the third category. Thus, there were deltaic areas. three categories in every rank (mansab).  The stimulus of contact to northern India was  The salary due to the soldiers was provided by traders, conquerors, and Jaina, added to the salary of the mansabdar, Buddhist, and some brahmana missionaries who was paid by assigning to him a who brought with them elements of material jagir. Sometimes, the mansabdars were culture from the north. paid in cash.   The origins of the mansabdari system can, This cultural and economic contact perhaps, he traced back to Changez Khan between the north and the deep south who organized his army on a decimal basis, is profoundly known Tamizhakam. the lowest unit of command being ten, and  The route to the south, called Dakshinapatha, the highest ten thousand (toman) whose was valued by the northerners because the commander was called khan. south supplied gold, pearls, and various  Persons were generally appointed at a low precious stones. mansab and gradually promoted, depending  This north-south connect is evident from upon their merits and the favor of the the writings of Megasthenes which mention emperor. A person could also be demoted as a mark of punishment. the Pandya state in the south.  The earlier Sangam texts are familiar with GS SCOREthe rivers Ganges and Son, and also with 71. Correct Option (d) Pataliputra, the capital of the Magadhan Explanation: empire.  Statement 1 is incorrect: Megaliths were  The Ashokan inscriptions mention the people living in the upland portions of the Cholas, Pandyas, Keralaputras, and southern peninsular India as early as 1000 Satyaputras living on the borders of the BC. empire and the Tamraparnis, or the people  Statement 2 is incorrect: These were of Sri Lanka. predominantly hunters and did not practice  The Brahmanical influence also percolated an advanced type of agriculture. in a large measure to Tamizhakam, but this  Statement 3 is incorrect: Tamizhakam really happened after the fourth century AD. was the cultural and economic contact Eventually, many elements of Tamil culture between the north and the Deep South spread to the north, and in the Brahmanical regions of India provided by traders, texts, the Kaveri came to be regarded as one conquerors, etc. of the holy rivers of India.

PTS2020/PPP-21/122019/26 32

Website ➡ https://upscpdf.com https://t.me/UPSC_PDF Website ➡ https://upscpdf.com https://t.me/UPSC_PDF

72. Correct Option: (c)  This religious text dealt with the Vaishnava teachings, as did the Vishnu Explanation: Purana, and also to an extent the Vishnu Smriti.  Option (c) is incorrect: The title of ‘Tuti- i-Hind’ was given to Amir Khusrau by  Bhagavatism or Vaishnavism Alauddin Khalji. overshadowed Mahayana Buddhism by Gupta times. Supplementary notes:  It preached the doctrine of incarnation, Alauddin Khalji or avatar. History was presented as a cycle of the ten incarnations of Vishnu.  Alauddin Khalji was Jalaluddin’s nephew and son-in-law.  A few Gupta kings were worshippers of Shiva, the god of destruction, but he  He treacherously murdered his father- came to the fore at a later stage, and does in-law Jalaluddin Khalji near Kara and not seem to have been as important as usurped the throne of Delhi. Vishnu in the early phase of the Gupta  Jalaluddin’s policy of tolerance was reversed rule. by Alauddin Khalji, who awarded drastic  Idol worship in the temples became a punishments to all those who dared to oppose common feature of Hinduism from the him. He decided to revive Balban’s Gupta period onwards policies of ruthless governance.  The Gupta kings followed a policy of  Alauddin adopted the title of Sikander- tolerance towards different religious i-Azam and gave Amir Khusrau the title sects. of Tuti-i-Hind (Parrot of India).  We find no example of the persecution of  He became the first ruler to fix land the followers of Buddhism and Jainism. revenue in cash.  This was also due to the change in the  His theory of kinship was secular and he character of Buddhism which had come proclaimed “Kingship knows no Kinship”. to acquire many features of Brahmanism He is known for his inclusion of non-Turks or Hinduism. in his service.  He refuted the suzerainty of the 74. Correct Option: (b) Caliph and did not allow any power independent of the state to guide his Explanation: policy.  Option (b) is correct Supplementary notes: 73. Correct option: (c) Dagh System Explanation:  During Akbar’s reign great care was taken to ensure that, the  Both statements are correct sawars recruited by the nobles were Supplementary notes: experienced and well-mounted. For this purpose, a descriptive roll (chehra) Religious developments during the Gupta of the soldier was maintained, and his Period horse was branded with the imperial marks. This was called the dagh  Bhagavatism originated in post-Maurya GS SCOREsystem. times and centred around the worship of Vishnu or Bhagavata.  Every noble had to bring his contingent for  periodic inspection before persons appointed Bhagavatism was marked by bhakti and by the emperor for the purpose. ahimsa. Bhakti meant the offer of loving devotion.  The horses were carefully inspected and only good quality horses of Arabic and Iraqi  The new religion was sufficiently liberal breed were employed. to attract foreigners. It also appealed  to artisans and merchants who became Provision was made that the contingents important under the Satavahanas and of the nobles should be mixed ones that is Kushans. drawn from all the group-Mughal, Pathan, Hindustani and Rajput. Thus, Akbar tried  Krishna taught in the Bhagavadgita to weaken the forces of tribalism and that even women, vaishyas, and shudras parochialism. Only the Mughal and Rajput who were born of sin could seek refuge nobles were allowed to have contingents in him. exclusively of Mughals or Rajputs.

33 PTS2020/PPP-21/122019/26

Website ➡ https://upscpdf.com https://t.me/UPSC_PDF Website ➡ https://upscpdf.com https://t.me/UPSC_PDF

75. Correct Option: (b) Afghan, Sher Shah Suri. It was fought on 26 June 1539 at Chausa, 10 miles Explanation: southwest of Buxar in modern-day Bihar,  Option (b) is correct India. Humayun escaped from the battlefield to save his life. Sher Shah was victorious Supplementary notes: and crowned himself the emperor. Satavahanas  Battle of Sirhind: The Battle of Sirhind  was fought between the Mughal The founder of this dynasty is known as Empire and the Suri Empire in 1555. Simuka and he ruled from 235 B.C. to After the death of Islam Shah Suri, the Suri 21.3 B.C. He was succeeded by his brother Empire was in a civil war where various Krishna. contenders to the throne fought each other  The fortunes of the family were for supremacy. Sikandar Shah Suri was restored by Gautamiputra Satakarni occupied with his struggle against Ibrahim (AD 106–30) who called himself the only Shah Suri when Humayun mobilized brahmana. He defeated the Shakas and an army from Kabul. On June 22, 1555, destroyed many kshatriya rulers. Humayun and Bairam Khan carried out a daring raid, defeated Sikandar Shah and  He claimed to have ended the Kshaharata reestablished their empire in India. lineage to which his adversary Nahapana belonged. This claim is true because over 8000 silver coins of Nahapana, found near 77. Correct Option: (b) Nasik, bear the marks of having been restruck by the Satavahana king. Explanation:   He also occupied Malwa and Kathiawar Option (b) is correct which were controlled by the Shakas. It Supplementary notes: seems that the empire of Gautamiputra Satakarni extended from Malwa in the Vaniggrama north to Karnataka in the south, and  Merchants in ancient India had a guild- he possibly also exercised general like professional organisationand their authority over Andhra. represenatives like Sresthi and Sarthavaha enjoyed a prominent position in the Gupta 76. Correct Option: (d) monarchical system.  Vaniggrama was a mercantile Explanation: organization in the Gujrat and deccan  Option (d) is correct region.Both the local merchants and non-local ones belonged to it. Supplementary notes:  It was a professional body of merchants Important Battles during Mughal Advent of different areas and dealing in different into India commodities.  First Battle of Panipat: The First Battle  The term should not be taken in its literal of Panipat was fought between the invading sense as the village of merchants. forces of Babur and the Lodi Empire, which took place on 21 April 1526 in North India. It marked the beginning of the Mughal 78. Correct Option: (a) Empire. This wasGS one of the earliest battlesSCORE Explanation: involving gun powder firearms and field artillery.  Option (a) is correct:  Battle of Khanwa: The Battle of Khanwa List I List II was fought near the village of Khanwa, in Bharatpur District of Rajasthan, on A. Diwan-i-Wizarat 4. Finance March 16, 1527. It was fought between the department invading forces of the first Mughal Emperor Babur and the Rajput forces led by Rana B. Diwan-i-Arz 3. Military Sanga of Mewar, after the Battle of Panipat. department The victory in the battle consolidated the new Mughal dynasty in India. C. Diwan-i-Rasalat 2. Department of religious affairs  Battle of Chausa: The Battle of Chausa was a notable military engagement between D. Diwan-i-Insha 1. Department of the Mughal emperor, Humayun, and the correspondence

PTS2020/PPP-21/122019/26 34

Website ➡ https://upscpdf.com https://t.me/UPSC_PDF Website ➡ https://upscpdf.com https://t.me/UPSC_PDF

Supplementary notes:  The famous king Karikala founded Puhar (coterminous Central Administration in the Sultanate with Kaveripattanam, the Chola Period capital) and constructed 160 km of  The office of the Sultan was the most embankment along the Kaveri River important in the Sultanate and he was the during the second century AD. ultimate authority for the military, legal,  They maintained an efficient navy. and political matters.  Pandya  The Sultan was assisted by a number of  ministers who headed different departments The Pandya territory occupied the and aided in administration. These ministers southernmost and the south-eastern were chosen by Sultan and they remained portion of the Indian peninsula with in office at his pleasure. Madurai in Tamil Nadu as its capital.   The post of Naib was the most powerful one. Sangam literature refers to the Pandya rulers. They were profited from trade The Naib practically enjoyed all the powers with the Roman empire and sent of the Sultan and exercised general control ambassadors to the Roman emperor over all the departments. Augustus.  Next to him, was the Wazir, who headed  The Brahmanas enjoyed considerable the finance department called Diwan-i- influence, and the Pandya king performed Wizarat. Vedic sacrifices in the early centuries of  The military department was the Christian era. called Diwan-i-Arz. It was headed  Chera by the Ariz-i-mumalik who was responsible for recruiting the soldiers  The Chera or the Kerala country was and administering the military situated to the west and north of the department. land of the Pandyas.  Diwan-i-Rasalat was the department  In the early centuries of the Christian of religious affairs, which dealt with era, the Chera state was as important pious foundations and gave stipends as the states of the Cholas and Pandyas to deserving scholars and men of piety. and owed its position to trade with the Grants were made by this department Romans. also for the construction and maintenance  The history of the Cheras is a continuing of mosques, tombs, and madrasas. It was battle with the Cholas and Pandyas. headed by chief Sadr who also functioned  as Chief Qazi, as the head of the judicial According to the Chera poets, their department. greatest king was Senguttuvan, who is said to invade the north and  The department of correspondence crossed the Ganges which seems an was called Diwan-i-Insha. All the exaggeration. correspondence between the ruler and the  sovereigns of other states as well as with his After the second century, Chera’s power subordinate officials was dealt. declined, and we know nothing of its history until the eighth century.

79. Correct Option (c) GS SCORE80. Correct Option: (a) Explanation: Explanation:  Both statements are correct  Option (a) is correct: Deva Raya II Supplementary notes: is known to have acquired the title of  Chola Gajabetegara or “Hunter of elephants”.  The Chola kingdom, which came to be Supplementary notes: called Cholamandalam (Coromandel), Deva Raya II in early medieval times, was situated to the north-east of the territory of the  Deva Raya II was the greatest of the Pandyas, between the Pennar and the Sangama dynasty rulers who was an able Velar rivers. administrator, an ambitious warrior, and a man of letters.  Their chief center of political power lay  at Uraiyur, a place famous for cotton He authored well-known works in the trade. Kannada language (Sobagina Sone and

35 PTS2020/PPP-21/122019/26

Website ➡ https://upscpdf.com https://t.me/UPSC_PDF Website ➡ https://upscpdf.com https://t.me/UPSC_PDF

Amaruka) and in the Sanskrit language Harappan civilization was not the Indus (Mahanataka Sudhanidhi). but the Saraswati River and its tributaries which flowed between the Indus and the  He also wrote a commentary on the Ganga. Brahmasutra.  He acquired the title of Gajabetegara, which literally means “Hunter of 82. Correct Option: (a) Elephants”, an honorific title that Explanation: explained his addiction to hunting elephants or a metaphor referring to  Statement 2 is incorrect: Under the his victories against enemies who were Nasaq system, the amount payable was as strong as elephants. calculated based on an assessment of what they have been paying in the past. 81. Correct Option: (b)  Statement 3 is incorrect: Under the Batai system, the produce was divided between Explanation: the peasants and state in fixed proportion.  Statement 2 is incorrect: As many as Supplementary notes: 1,100 (80%) settlements are located on the vast plain between the Indus and the Land Revenue Administration under Ganga, comprising mainly the Saraswati Akbar river system which is dry today.  The most common and the oldest was Supplementary notes: called batai or ghalla-bakhshi. In this system, the produce was divided Harappan Civilization: Geographical between the peasants and the state in Distribution fixed proportion. The crop was divided after it had been thrashed; or when it had  The Harappan civilization was discovered been cut and tied in stacks, or while it was in 1920-21 following the excavations by standing in the field. R.D. Banerjee at Mohenjodaro and by D.R. Sahni at Harappa.  In 1580, Akbar instituted a new system called the dahsala. Under this system,  The 1400 settlements, discovered so the average produce of different far are distributed over a very wide geographical area. Its known extent crops as well as the average prices in the west is upto Sutkagendor in prevailing over the last ten (dah) years Baluchistan; Alamgirpur in Meerut were calculated. One third of the average District (Uttar Pradesh) in the east; produce was the state share. The state Daimabad (Ahmadnagar District, demand was, however, stated in cash. This Maharashtra) in south; and Manda was done by converting the state share into (Akhnoor District, Jammu and money on the basis of a schedule of average Kashmir) in the north, covering an area prices over the past ten years. of almost 1600 km. east-west and 1400 km.  A third system which was widely used north-south. in Akbar’s time was nasaq. It was  These settlements are mostly located on a rough calculation of the amount river banks. payable by the peasant on the basis of what he had been paying in the past.  When we look at the distribution pattern of these settlements in terms of rivers, we find that: GS SCORE83. Correct Option: (c)  Only 40 settlements are located on the Explanation: Indus and its tributaries  Both statements are correct  As many as 1,100 (80%) settlements are located on the vast plain Supplementary notes: between the Indus and the Ganga, Family Structure during Satavahanas comprising mainly the Saraswati Phase river system which is dry today  The most interesting detail about the  About 250 settlements are found in Satavahanas relates to their family India beyond the Saraswati river system structure. In the Aryan society of north a number of them in Gujarat, and a few India, the father enjoyed greater importance in Maharashtra. than the mother, and the north Indian  It is clear from the above distribution princes generally belonged to a patriarchal pattern of settlements that the focus of society.

PTS2020/PPP-21/122019/26 36

Website ➡ https://upscpdf.com https://t.me/UPSC_PDF Website ➡ https://upscpdf.com https://t.me/UPSC_PDF

 The Satavahanas however show traces 85. Correct Option: (a) of a matrilineal social structure. It was customary for their king to be named after Explanation: his mother. Such names as Gautamiputra  Option (a) is correct: and Vashishthiputra indicate that in their society the mother enjoyed a great deal of List I List II importance. A. Arasar 2. Ruling class  Sometimes an inscription is issued both B. Melkannakku 3. Eighteen Major Works under the authority of the king and his of Sangam Literature mother. C. Tirukkural 4. Tamil text deals with  At present in peninsular India, the son’s philosophy and wise name includes a part of the father’s name, maxims and there is no place in it for the mother’s, indicative of patriarchal influence. D. Silappadikaram 1. Love story of Kovalan and a courtesan  Queens made important religious gifts in Madhavi of their own right, and some of them acted as Kaveripattanam regents.  However, the Satavahana ruling family was Supplementary notes: basically patriarchal because succession to Tamil Societal Setup the throne passed to the male member.  The Brahmanas first appear in the Tamil land in the Sangam age. 84. Correct Option: (b)  The Tamil Brahmanas partook of meat and Explanation: wine. The kshatriyas and vaishyas appear as regular varnas in the Sangam texts.  Statement 2 is incorrect: Iltutmish was the first to issue purely Arabic coinage of  The warrior class was an important element silver tanka, which remains the basis of the in the polity and society. The captains of the modern rupee and copper Jital. army were invested with the title of “Enadi” at a formal ceremony. Supplementary notes:  Civil and military offices were held under Iltutmish both the Cholas and Pandyas by “Vellalas”  Iltutmish is known to be the real consolidator or rich peasants. of Turkish rule in India.  The ruling class was called “Arasar”, and  He belonged to the Ilbari tribe and hence his its members had marriage relations with dynasty also known as the Ilbari dynasty. the vellalas, who formed the fourth caste.  He was the son-in-law of Qutbuddin Aibak  Agricultural operations were generally and disposed of Aram Shah (son of the task of members of the lowest class or Qutbuddin Aibak) and became the Sultan “Kadaisiyar”. with the name of “Shamsuddin”. Tamil Language and Sangam Literature  After becoming Sultan, he shifted his capital  from Lahore to Delhi. Sangam was a college or assembly of Tamil poets held probably under the patronage of  He divided his empire into Iqtas which the chiefs or kings. included the assignment of land in lieu of GS SCORE salary. Every Iqtadars had to maintain law The available Sangam literature, which was and order and collect revenue. produced by these assemblies, was compiled in AD 300–600.  “Turkan-i-chahalgani” was organized by him as a new class of ruling elite  The Sangam literature can roughly be of forty powerful military leaders, the divided into two groups, narrative and Forty. didactic.  These were Turkish amirs (nobles)  The narrative texts are called who advised and helped the Sultan in “Melkannakku” or Eighteen Major administering the Sultanate. The group Works. They comprise eighteen major was finally eliminated by Balban. works consisting of eight anthologies and ten idylls.  He was the first to issue purely Arabic coinage of silver tanka weighing 175  The didactic works are called grams, which remains the basis of the “Kilkanakku” or Eighteen Minor modern rupee and copper Jital. Works.

37 PTS2020/PPP-21/122019/26

Website ➡ https://upscpdf.com https://t.me/UPSC_PDF Website ➡ https://upscpdf.com https://t.me/UPSC_PDF

 Besides the Sangam texts, we have a text  Their contribution to literature includes: called Tolkkappiyam, which deals with  Manucharitam and Harikathasaram grammar and poetics. by Allasani Peddanna (the greatest,  Another important Tamil text deals with who was also called Andhrakavita philosophy and wise maxims and is called Pitamaga). Tirukkural.  Panduranga Mahamatyam, authored  The other important twin Tamil epics are by Tenali Ramakrishna. Silappadikaram and Manimekalai.  Pingali Suranna, author of The two were composed around the sixth Garuda Puranam, Prabhavatee century. Pradyumnamu, Raghava Pandaveeyam and Kalapurnodayamu.  The first is considered to be the brightest gem of early Tamil literature. It deals with a love story 87. Correct Option: (b) in which a dignitary called Kovalan prefers a courtesan called Madhavi Explanation: of Kaveripattanam to his wedded  Statement 1 is incorrect: He was not a wife Kannagi from a noble family. staunch Hindu Ruler.  The other epic, Manimekalai, was  Statement 2 is incorrect: His authority written by a grain merchant of Madurai. was limited to north India excluding It deals with the adventures of the Kashmir. daughter born of the union of Kovalan Supplementary notes: and Madhavi. However, this epic is of greater religious than literary interest. Harsha’s Kingdom  Harsha is called the last great Hindu 86. Correct Option: (d) emperor of India, but he was neither a staunch Hindu nor the ruler of the Expanation: whole country.  Option (d) is correct:  His authority was limited to north India List I List II excluding Kashmir. Rajasthan, Punjab, UP, Bihar, and Orissa were under his direct control, but his sphere of influence A. Jambavati 4. Krishna Deva spread over a much wider area. Kalyanam Raya  It appears that the peripheral states B. Manucharitam 3. Allasani acknowledged his sovereignty. In eastern Peddanna India he faced opposition from the Shaivite king Shashanka of Gauda, who felled C. Panduranga 2. Tenali the Bodhi tree at Bodh-Gaya. However, Mahamatyam Ramakrishna Shashanka’s death in AD 619 put an end to this hostility. D. Garuda Puranam 1. Pingali  Suranna Harsha’s southward march was stopped at the Narmada River by the Supplementary notes: Chalukya king Pulakeshin, who ruled over a great part of modern Karnataka Literature during the Vijayanagar and Maharashtra with his capital at period GS SCOREBadami in the modern Bijapur district of Karnataka.  Krishna Deva Raya was great patron of literature and art.  Apart from this, Harsha did not face any serious opposition and succeeded in giving  He himself authored: a measure of political unity to a large part  Amukthamalyadha (epic poem in of India. Telugu)  Jambavati Kalyanam (Sanskrit 88. Correct Option: (a) drama based on the Jambavati, Explanation: one of the eight principal queen- consorts of Lord Krishna.)  Pair 2 and 3 are incorrectly matched:  Amir’s were those who held ranks from 500 Ushaparinayam (Sanskrit drama). to below 2500 and those holding ranks of  He adorned eight eminent scholars of Telugu 2500 and above were called Amir-i-Umda or known as the ‘Ashta diggajas’ in his court. Umda-i-Azam.

PTS2020/PPP-21/122019/26 38

Website ➡ https://upscpdf.com https://t.me/UPSC_PDF Website ➡ https://upscpdf.com https://t.me/UPSC_PDF

Supplementary notes:  The Rig Veda is the earliest text of the Indo-European languages. It is written in Mansabdari System Sanskrit but also includes many Munda  Under this system, every officer was and Dravidian words. Probably these assigned a rank (mansab). The ranks or words percolated to the Rig Veda through mansabs were divided into two-zat and the language of the Harappans. sawar.  It is a collection of prayers offered to Agni,  The word zat means personal status of a Indra, Mitra, Varuna, and other gods by person and also the salary due to him. The various families of poets or sages. It consists sawar rank indicated the number of cavalry of mandalas or books, of which books II to men (sawars) a person was required to VII forms the earliest parts. Books I and X seems to have been the latest additions to maintain. it.  A person who was required to maintain as  The earliest Aryans lived in the geographical many sawars as his zat rank was placed area covered by eastern Afghanistan, the in the first category of zat rank; if he North-West Frontier Province, Punjab, and maintained half or more, then in the second the fringes of western UP. category and if he maintained less than half then in the third category. Thus, there were  The entire region in which the Aryans first three categories in every rank (mansab). settled in the Indian subcontinent is called the Land of the Seven Rivers.  Persons holding ranks below 500 zat were called mansabdars, those from  The Sindhu, coterminous with the Indus, is 500 to below 2500 were called amirs, the river par excellence of the Aryans, and and those holding ranks of 2500 and it is repeatedly mentioned. Another river, above were called amir-i-umda or the Sarasvati, is called naditama or umda-i-azam. the best of rivers in the Rig Veda. It is identified with the Ghaggar–Hakra channel  However, the word mansabdar is sometimes in Haryana and Rajasthan, but its Rig Vedic used for all the three categories. description shows it to be the Avestanriver  Apart from status, this classification had Harakhwati or the present Helmand river significance: an amir or an amir-i-umda in south Afghanistan from where the name could have another amir or mansabdar Sarasvati was transferred to India. serve under him, but a mansabdar could not do so. Thus, a person with a rank of 5000 90. Correct Option: (c) could have under him a mansabdar up to a rank of 500 zat, and one with a rank of4000 Explanation: could have a mansabdar up to a rank of 400  zat, and so on. Both statements are correct Supplementary notes: 89. Correct Option: (d) Village Assemblies During Chola Explanation:  There were two assemblies, called the Ur and the Sabha or Mahasabha.  Statement 1 is incorrect: Sarasvati is called naditama or the best of the rivers  The Ur was a general assembly of the in the Rig Veda. village.  Statement 2 is incorrect: Rigveda is  Mahasabha was a gathering of the GS SCOREadult men in the Brahman villages written in Sanskrit, and it also includes many Munda and Dravidian words. which were called Agraharams. These were villages with Brahman settlements in Supplementary notes: which most of the land was rent-free. RigVeda  These villages enjoyed a large measure of autonomy. The affairs of the village  The Rigveda represents the earliest sacred were managed by an executive book of India. It is the oldest and biggest committee to which educated persons amongst all the four Vedas. In it, we find the owning property were elected either seeds of India’s religious and philosophical by drawing lots or by rotation. development.  These members had to retire every three  We know about the Aryans in India from the years. There were other committees for Rig Veda. The term arya generally denotes helping in the assessment and collection of a cultural community that speaks the Indo- land revenue, for maintenance of law and Aryan language. order, justice, etc.

39 PTS2020/PPP-21/122019/26

Website ➡ https://upscpdf.com https://t.me/UPSC_PDF Website ➡ https://upscpdf.com https://t.me/UPSC_PDF

 One of the important committees was the  Almost every house had a bathroom, and in tank committee which looked after the some cases there is evidence of bathrooms distribution of water to the fields. The on the first floor. Mahasabha could distribute new lands, and  The most striking feature of Harappan exercise ownership rights over them. civilization is the advanced drainage system.  It could also raise loans for the village and Small drains ran from each house levy taxes. The self-government enjoyed by and were connected to larger drains these Chola villages was a very fine system. running alongside the main roads. The To some extent this system worked in the drains were loosely covered to allow other villages as well. regular cleaning and maintenance. The importance placed on health and hygiene – 91. Correct Option: (b) both public and private is quite impressive.

Explanation: 92. Correct Option: (b)  Statement 2 is incorrect: Higher citadel in west and lower town in east. Explanation:   Statement 4 is incorrect: Doorways and Statement 1 is incorrect: The directly windows rarely opened out into the main administered territories in the Pala and street, but faced side lanes to protect from Pratihara empires were divided into bhukti the view of the passers-by. and mandala. Supplementary notes: Supplementary notes: Harappan Town Planning Pala and Pratiharas  The Harappan culture was distinguished  The directly administered territories by its system of town planning. The overall in the Pala and Pratihara empires layout of the Indus-Saraswati cities is were divided into bhukti (provinces), distinguished by the orientation of streets and mandala or visaya (districts). and buildings, according to the cardinal  The governor of a province was called uparika directions - east-west, and north-south and the head of a district, visayapati. The thus forming a grid system, with roads uparika was expected to collect land cutting across one another virtually revenue and maintain law and order at right angles. Even the width of these with the help of the army. The visayapati streets was in a set ratio. was also expected to do the same within his  Both Harappa and Mohenjodaro had a jurisdiction. During the period, there, was higher western citadel or acropolis, and an increase of smaller chieftains, called this was possibly occupied by members samantas or bhogapatis, who dominated of the ruling class. Below the citadel in over a number of villages. each city lay a lower town in the east  The visayapatis and these smaller chiefs with brick houses that were inhabited by the common people. tended to merge with each other, and later on the word samanta began to be used  The use of burnt bricks in the indiscriminately for both of them. Harappan cities is remarkable because  in the contemporary buildings of Egypt In the Rashtrakuta kingdom, the dried bricks were primarily used. We find directly administered areas were the use of bakedGS bricks in contemporary SCORE divided into rashtra (provinces), visaya Mesopotamia, but they were used to a and bhukti. The head of rashtra was much larger extent in the Harappan cities. called rashtrapati, and he performed The doors windows were made of wood and the same functions as the Uparika did mats. in the Pala and Pratihara empires.   Doorways and windows rarely opened The visaya was like a modern district, and out into the main street, but faced side the bhukti was a smaller unit to it. In the lanes. The view into the house was Pala and Pratihara empires, the unit below blocked by a wall or a room around the the visaya was called pattala. front door. This was done to protect  The precise role of these smaller units is not the activities in the central courtyard known. It seems that their main purpose from the view of passers-by. was the realization of land revenue and  Many houses were at least two storied and some attention to law and order. Apparently some scholars think that some of the houses all the officials were paid by giving them may have been three storied. grants of rent-free land.

PTS2020/PPP-21/122019/26 40

Website ➡ https://upscpdf.com https://t.me/UPSC_PDF Website ➡ https://upscpdf.com https://t.me/UPSC_PDF

 This tended to blur the distinction between Supplementary notes: local officials and the hereditary chiefs and smaller vassals. Similarly, the rashtrapati Krishna Deva Raya (1509–29) or governor sometimes enjoyed the status  He was the greatest of the Vijayanagar and title of a vassal king. rulers, also known as ‘Abhinava Bhoja’, ‘Andhra Pitamah’, ‘Andhra Bhoja’ (as he 93. Correct Option (a) was great patron of literature and art).  Explanation: He invaded the Raichur Doab and captured the city of Raichur in c.1520 CE.  Option (a) is incorrect: Arts and crafts  in particular witnessed remarkable growth He restored the to during this period. Muhammad Shah, which was taken over by independent kingdoms of Deccan Sultanate Supplementary notes: and Krishna Deva himself took the title of Yavanarajya sthapanacharya. Trade and craft during 200 BC to AD 250   The age of the Shakas, Kushans, was the collective title Satavahanas (200 BC-AD 250) and the given to the eight Telugu poets in his first Tamil states was the most flourishing court of the emperor Sri Krishna Deva period in the history of crafts and commerce Raya. in ancient India.  The age of Ashtadiggajas is called Prabandha  Arts and crafts, in particular, witnessed Age (1540 AD to 1600). remarkable growth.  These poets were:  The Digha Nikaya, which relates to  Allasani Peddanna pre-Maurya times, mentions nearly two dozen occupations, but the Mahavastu,  which relates to this period, catalogs  Madayyagari Mallana thirty-six kinds of workers living in the town of Rajgir, and the list is not  Dhurjaty exhaustive.  Ayyalaraju Ramambhadrudu  The Milinda Panho or the Questions of  Milinda enumerates as many as seventy-five Pingali Surana occupations, sixty of which are connected  Ramarajabhushanudu with various crafts.  Tenali Ramakrishna  A Tamil text known in English as ‘The Garland of Madurai’ supplements the  The most famous being Allasani information supplied by the two Buddhist Peddana honoured with the title texts on crafts and craftsmen. ‘Andhrakavitapitamaha’.  Many artisans and merchants were  is an epic poem in organized into guilds called sreni and Telugu composed by ayatana. of the Vijayanagar Dynasty. Amuktamalyada translates to “A  Both merchants and craftsmen were divided into high, low, and middle ranks. garland of pearls”.   The Buddhist texts mention the “Sresthi”, Considered as a masterpiece, who was the chief merchant of the Nigama, Amuktamalyada describes the story of GS SCOREwedding of the Hindu Lord Vishnu and and the “Sarthavaha”, the caravan leader who was the head of the corporation of Andal - the Tamil Alvar poet and daughter merchants (vanijgramo). of Periyalvar, at Srirangam.  It also speaks of nearly half a dozen petty merchants called “Vanija”. They dealt with 95. Correct Option: (d) fruits, roots, cooked food, sugar, barkcloth, sheaves of corn or grass, and bamboo. Explanation:  The term “Vyavahari”, that is one who  Option (d) is correct transacts business. Supplementary notes:

94. Correct Option: (d) Rigvedic Polity  The polity of the Rigvedic India was Explanation: basically a tribal polity with the tribal chief  All statements are correct in the center.

41 PTS2020/PPP-21/122019/26

Website ➡ https://upscpdf.com https://t.me/UPSC_PDF Website ➡ https://upscpdf.com https://t.me/UPSC_PDF

 There was a continuous struggle for 97. Correct Option: (b) supremacy among different kings and republics chief such as the Battle of ten Explanation: Kings. Tribal struggle was a part of the  Statement 2 is incorrect: The Gupta evolutionary process towards the formation kingdom was enlarged enormously of a larger political entity. by Chandragupta’s son and successor  The political structure of Rigvedic Samudragupta. India may be traced in the following Supplementary notes: ascending order:  The Family (kula) Gupta Period   The Village (grama) After the break-up of the Maurya empire, the Satavahanas and Kushans emerged as  The Clan (vis) the two large powers in the southern and  The People (Jana) northern regions of India.   The Country (Rashtra) The Kushan power in north India came to an end around AD 230, and Murundas,  Kula (family) was the smallest unit. It who were possibly kinsmen of the Kushans included all the people living under the continued to rule till AD 250. same roof (griha).  Twenty-five years later, in about AD 275,  An aggregate of several families or Kulas the Gupta dynasty came to power. made up the grama and its headman was  called gramini. The Guptas were possibly feudatories of the Kushans in UP, and seem to have succeeded  The next larger formation was called the them without any considerable time-lag. vis, under the head called vispati.  The first important king of the Gupta  Larger than vis was Jana. The tribe was dynasty was Chandragupta I. He married a called Jana for e.g. tribes like Yadus, (Yadva- Lichchhavi princess, in all probability from janaha) and Bharatas and the tribal chief Nepal, which strengthened his position as was called Rajana. The king is also called the Guptas were probably vaishyas, and the protector of the Jana or people. hence marriage into a kshatriya family lent them prestige.  Above the jana in the order was rashtra. The kingdoms (Rashtra) were generally  Chandragupta, I started the Gupta era small states ruled by kings (Rajana). in AD 319–20, which marked the date of his accession. 96. Correct Option: (b)  The Gupta kingdom was enlarged enormously by Chandragupta’s son and Explanation: successor Samudragupta (AD 335–80).  Option (b) is correct: The correct order is  Harishena the famous court poet of Muqaddam - Amil - Shiqdar – Muqti. Samudragupta has written a glowing account of the military exploits of his patron Supplementary notes: and countries that were conquered by him. Provincial Government in the Sultanate He is also known as the Napoleon of India. Period  The provinces under the Delhi Sultanate 98. Correct Option: (c) were called iqtas. They were headed by the Governor who wasGS called “Muqtis or walis”.SCORE Explanation: They were to maintain law and order and  Statement 1 is incorrect: This empire collect land revenue. was founded by Gopala.  The provinces were further divided into Supplementary notes: shiqs (districts), which was under the control of the “Shiqdar”. Pala Empire  The next division subdivision was pargana  The Pala empire was founded by (sub-district), comprising a number of Gopala, probably in AD 750 when he was villages and was headed by the “Amil”. elected king by the notable men of the area to end the anarchy prevailing there.  The villages were grouped into units of 100 or 84 (traditionally called chaurasi).  Gopala was not born in a high, much less The village remained the basic unit of the a royal family, his father probably was a administration. The village headman was soldier. He unified Bengal under his control, known as “Muqaddam or Chaudhri”. The and even brought Magadha (Bihar) under village accountant was called ‘Patwari’ his control.

PTS2020/PPP-21/122019/26 42

Website ➡ https://upscpdf.com https://t.me/UPSC_PDF Website ➡ https://upscpdf.com https://t.me/UPSC_PDF

 Gopala was succeeded in AD 770 by his son, 100. Correct Option: (a) Dharamapala, who ruled till AD’810. Explanation:  His reign was marked by a tripartite struggle between the Palas, the  Statement 2 is incorrect: The Afghan Pratiharas and the Rashtrakutas for forces led by Hemu were defeated at the 2nd the control of Kanauj and north India. Battle of Panipat in 1556 by the Mughals.  The Pratihara ruler advanced upon Gaud Supplementary notes: (Bengal), but before a decision could be Akbar Ascendance to the Throne taken, the Pratihara ruler was defeated by the Rashtrakuta ruler, Dhruva, and  When Humayun fled to Iran, young Akbar was forced to seek refuge in the deserts of was captured by his uncle, Kamran. He Rajasthan. Dhruva then returned to the treated the child well on the whole. Akbar Deccan. was re-united with his parents after the capture of Qandhar.  This left the field free for Dharmapala who occupied Kanauj and held a grand darbar  When Humayun died, Akbar was at Kalanaur which was attended by vassal rulers from in Punjab, commanding operations against Punjab, eastern Rajasthan, etc. the Afghan rebels there. He was crowned at Kalanaur in 1556 at the young age of thirteen years and four months. 99. Correct Option: (a)  Akbar succeeded to a difficult position. The Explanation: Afghans were still strong beyond Agra, and were regrouping their forces under the  Option (a) is correct leadership of Hemu for a final showdown. Supplementary notes: Kabul had been attacked and besieged. Sikandar Sur, the defeated Afghan ruler, Harsha and His times was loitering in the Siwalik Hills.  The early history of Harsha’s reign is  However, Bairam Khan, the tutor of reconstructed from a study by Banabhatta, Akbar and a loyal and favorite officer who was his court poet and who wrote a of Humayun, rose to the occasion. He book called Harshacharita. became the wakil of the kingdom, with  This can be supplemented by the account the title of Khan-i-Khanan, and rallied of the Chinese pilgrim Hsuan Tsang, the Mughal forces. who visited India in the seventh century  The battle between the Mughals and the and stayed in the country for about fifteen Afghan forces led by Hemu, took place years. once again at Panipat (5 November  Harsha’s inscriptions speak of various types 1556) and the Afghans led by Hemu of taxes and officials. were defeated by Mughals. Thus, Akbar had virtually to reconquer his empire. GS SCORE™™™™™

43 PTS2020/PPP-21/122019/26

Website ➡ https://upscpdf.com